Docsity
Docsity

Prepare for your exams
Prepare for your exams

Study with the several resources on Docsity


Earn points to download
Earn points to download

Earn points by helping other students or get them with a premium plan


Guidelines and tips
Guidelines and tips

NR511 MIDTERM BEST EXAM PRACTICE REVIEW STUDY GUIDE LATEST UPDATE 2022/2023, Exams of Nursing

NR511 MIDTERM BEST EXAM PRACTICE REVIEW STUDY GUIDE LATEST UPDATE 2022/2023

Typology: Exams

2022/2023

Available from 12/28/2022

Expertsolution
Expertsolution 🇺🇸

3.8

(11)

2.9K documents

1 / 196

Toggle sidebar

Related documents


Partial preview of the text

Download NR511 MIDTERM BEST EXAM PRACTICE REVIEW STUDY GUIDE LATEST UPDATE 2022/2023 and more Exams Nursing in PDF only on Docsity! NR511 MIDTERM BEST EXAM PRACTICE REVIEW STUDY GUIDE LATEST UPDATE 2022/2023 • Actinic keratosis: pre-cancerous lesion. The main assessment technique is INSPECTION, which will show as flesh colored, hard and sand paper like. ▪ TX: cryotherapy o Risk factor: sun exposure, can progress to squamous cell carcinoma o Referrer pt to dermatology to prevent progression • Fungal skin infection: assess rash and satellite lesions. o DX: based on clinical presentation, most common is candida albicans o Tx: antifungal cream, pills, keep area as dry as possible. The fungus likes moisture and poor air circulation o At risk: opportunistic, pts who are immunocompromised, older and younger pts, diabetics, and antibiotic therapy. o Refer patient if there's no improvement • Common types of fungal infections: o Tinea vesicolor: flat to slightly elevated brown papules and plaques that scale when they are rubbed along with areas of hypopigmentation, pruritic, most commonly found on trunk and shoulders. o Balanitis: candidiasis in the glands of the penis o Tinea corporis: annual lesions with scaly borders and central clearing on the trunk o Tinea pedis: athlete's foot, and between toes o Tinea cruis: jock-itch groin • Bacterial skin infections: warm, red, painful w/o sharply demarcated border o Cellulitis: is a spreading infection of the epidermis and sub-cut tissue that usually begins after a break in the skin. o Folliculitis: bacterial infection of the hair follicle, papules are characteristics of folliculitis • Viral skin infections o Erythema infectiosum (fifth disease) erythematous, warm rash, gives the appearance of slapped cheeks. Sore throat, slight fever, upset stomach, headache, fatigue, and itching. Resolves on its own. o Varicella rash: contagious 48 hours before the onset of the vesicular rash, during the rash formation and during the several days it takes the vesicles to dry up. Characteristics rash appears 2-3 weeks after exposure. o Warts: caused by the human papillomavirus, most warts recur despite treatment. Contrary to popular opinion, warts do not have roots, the underside of a wart is smooth and round. Abrading the skin can spread the virus, vigorous rubbing, shaving, and nail biting can do the same. • Skin inflammations: o Pityriasis rosea : common, self-limiting, usually asymptomatic eruption with a distinct initial lesion. This "HERALD PATCH", which appears suddenly and without symptoms, usually is on the chest or back. ▪ Secondary lesions appear 1-2 wks later while the herald patch remains. ▪ The collarette scaling is another classic symptom of pityriasis rosea. NR511 MIDTERM BEST EXAM PRACTICE REVIEW STUDY GUIDE LATEST UPDATE 2022/2023 NR511 MIDTERM BEST EXAM PRACTICE REVIEW STUDY GUIDE LATEST UPDATE 2022/2023 ▪ The lesions usually resolve suddenly in 4-12wks w/o scarring. ▪ Outbreaks are known to occur in close quarters like military barracks or dormitories. o Hives: look at the location of the rash, the first step is to determine the need for epinephrine. Look for respiratory symptoms, SOB, hoarseness, look at location. If the rash is on the neck, face- admin epinephrine. o Cholinergic urticaria: hives or wheals that are pruritic and occur on the trunk and arms following exercise, anxiety, elevated body temp. hot bath and showers. ▪ Hx taking about when the rash started is important for dx o Contact dermatitis: poison ivy: a form of contact dermatitis, it is not contagious and it cannot be spread from one area of the body to another by touching it. Type of SPORE reaction. ▪ Another type is Latex sensitivity o Keratosis pilaris: mild pruritic and looks like GOOSEFLESH, the rash appears as small, pinpoint, follicular papules on a mildly erythematous base. It is a benign conditions that resolves by adulthood. o Atopic Dermatitis: consider ALLERGY!! ▪ Atopic triad: ASTHMA, ECZEMA, ALLERGIC RHINITIS ▪ RAST may be done to ID the antigen-specific mast cell activation or to quantify levels of antigen-specific IgE. RAST is usually available to PCPs, where as scratch testing is usually done by allergists. ▪ RAST results requires specialized knowledge, and should be used as general atopic screening tool. • Hair loss o Alopecia areata: systemic cause of alopecia, nonscarring hair loss of rapid onset, the pattern of which is most commonly sharply defined round or oval patches. o Trichotilomania: non-scarring, non-systemic causes of alopecia include trauma, bacterial or local fungal infections, and radiation to the head. o Minoxidil(Rogaine) vasodilator and may stimulate vertex hair growth. • Parasitic skin infections o Pediculosis: (LICE!!) client education is important in the tx of pediculosis b/c pts should be informed that itching may cause for up to a week after successful tx b/c of the slow resolution of the inflammatory reaction caused by the lice infestation. • Ear disorders o Otitis externa: classic sign of acute otitis externa is tenderness on traction of the pinna and/or pain on applying pressure over the tragus. There is typically an erythematous ear canal, and usually a hx of recent swimming. ▪ Using ear drops made of a solution of equal parts alcohol and vinegar in ea. Ear after swimming is effective in drying the ear canal and maintaining an acidic environment, therefore preventing a favorable medium for the growth of bacteria, the cause of swimmer's ears. o Acute otitis media: ear infection that is dx by otoscopic examination. The tympanic membrane will appear red and bulging with or w/o visible effusion. NR511 MIDTERM BEST EXAM PRACTICE REVIEW STUDY GUIDE LATEST UPDATE 2022/2023 NR511 MIDTERM BEST EXAM PRACTICE REVIEW STUDY GUIDE LATEST UPDATE 2022/2023 are best treated with supportive care. Antibiotics are ineffective and increase the risk of antibiotic resistance. o Peritonsillar Abscess: peritonsillar cellulitis and abscess are acute pharyngeal infections most common among adolescents and young adults. ▪ Infection is virtually always unilateral and is located btw the tonsil and the superior pharyngeal constriction muscle. ▪ S/S: gradual onset of severe unilateral sore throat, odynophagia, fever, otalgia, and asymmetric cervical adenopathy. Trismus, similar to lock jaw or "hot potato" voice (speaking as if a hot object was in the mouth), is common. A toxic appearance (e.g. poor or absent eye contact, failure to recognize parents, irritability, inability to be consoled or distracted, drooling, severe halitosis, tonsillar erythema, and exudates can also be observed. ▪ S/S cont: in pt's peritonsillar abscess, there is more of a discrete bulge, with deviation of the soft palate and uvula. Patients should be referred to the ED immediately as maintaining airway patency and preventing sepsis is of concern. o Candidiasis: painless, white, slightly raises patches on the pt's mouth, AKA thrush o Oral Cancer: 90% of oral cancers are squamous cell carcinoma (SCC) and is seen typically on the lip or lateral part of the toungue, usually as a lesion that is white, red, or mixed white and red. ▪ Characterized by painless, firm lesions with indurated borders. o Rhinitis (Hay Fever): allergic rhinitis results from immunoglobulin E(IgE)-mediated type I hypersensitivity to airborne irritants affecting eyes, nose, sinuses, throat, and bronchi. ▪ The s/s are allergic rhinitis are similar to those of viral rhinitis but usually persist and are seasonal in nature. When assessing the nasal mucosa, you will observe that the turbinates are usually pale or violaceous because of venous engorgement. • GI DISORDERS o Appendicitis: constant periumbilical pain shifting to the right lower quadrant, vomiting following the pain, a small volume of diarrhea. ▪ Fecalith: most common cause: stone made of feces typically found in the colon ▪ No systemic symptoms: headaches/ malaise/ myalgia ▪ A mild elevation of WBC count w/ early left shift, and WBCs or RBCs in the urine indications of appendicitis. ▪ The WBC count becomes high only with gangrene or perforation of the appendix. The urine may have EBCs or RBCs if the bladder is irritated and ketonuria if there is prolonged vomiting. ▪ Obturator sign: elicited when, with the patient's right hip and knee flexed. The examiner slowly rotates the right leg internally, which stretches the obturator muscle. Pain over the right lower quadrant is considered a positive sign. o Inflammatory bowel disease: Chron disease shows transmural inflammation, granulomas, focal involvement of the colon with some skipped areas, and sparing of the rectal mucosa, the inflammation extends deeper into the intestinal wall. NR511 MIDTERM BEST EXAM PRACTICE REVIEW STUDY GUIDE LATEST UPDATE 2022/2023 NR511 MIDTERM BEST EXAM PRACTICE REVIEW STUDY GUIDE LATEST UPDATE 2022/2023 ▪ Chron's disease can involve all or any layer of the bowel wall and any portion of the GI tract from the mouth to the anus. ▪ Any portion of the GI tract can be affected by 80% of pts have small bowel involvement. ▪ In advance disease, perianal lesions, fistulas, strictures, and obstructions are common in CD. ▪ Folic acid and serum levels of most vitamins, including A, B complex, C, and the fat- soluble vitamins, are decreased in CD as a result of malabsorption. Liver enzymes are increases. ▪ Ulcerative colitis is a disease only of the colon. While it is not the first tx of choise, total colectomy is tx option that can completely resolve this problem. ▪ In ulcerative colitis, the mucosal surface of the colon is inflamed. This ultimately results in friability, erosions, and bleeding. It most often occurs in the rectosigmoid areas but can involve the entire colon. ▪ In CD, patients with UC are more at risk for colon perforation and should be followed closely by surgeon. o GERD: Symptoms occur at night w/ regurgitation, heartburn is classic for GERD (mild to severe). Dysphagia is frequently a prominent symptom of GERD. ▪ It is usually associated with other symptoms, including regurgitation, water brash (reflex salivation) sour taste in the mouth in the morning, odynophagia, blenching, coughing, hoarseness, or wheezing, usually at night. ▪ It a patient has been tx w/ diet modifications and 6wks of omeprazole w/o improvement of symptoms, the next step is an endoscopy. ▪ A biopsy can be done and sent for H pylori at that time. ▪ Diverticular disease has been shown to be significantly increased with a low fiber diet and diet that is high in fat and red meats. Obesity is associated with a higher risk for diverticular disease. ▪ Clients w/ GERD should be instructed to avoid coffee, alcohol, chocolate, peppermint, and spicy foods. Eat smaller meals, smoking cessation, remain upright for 2 hours after meals, elevate the head of the 6-8 inches in blockers and refrain from eating for 3 hours before going to bed. o IBS: Tx: anti-diarrheals (Lomotil), laxatives, antispasmodics (Bentyl), tricyclic antidepressants, SSRIs (prozac), and medications to increase the intestinal fluid secretion and improve fecal transit (Linzess) are all used to treat IBS. o Acute diverticulitis: pts may have it with bleeding not associated with pain or discomfort. When diverticula become inflamed, there are usual s/s of infection: fever, chills, and tachycardia. Patients typically present with localized pain and tenderness in the LLQ of the abdomen with associated anorexia, nausea and vomiting. ▪ CT scan w/ contrast may sometimes be done to r/o if the GN etiology such as ovarian cyst of tumor as well as bowel pathology such as abdominal abscess. NR511 MIDTERM BEST EXAM PRACTICE REVIEW STUDY GUIDE LATEST UPDATE 2022/2023 NR511 MIDTERM BEST EXAM PRACTICE REVIEW STUDY GUIDE LATEST UPDATE 2022/2023 o Gastroenteritis: BOTULISM: GI illness associated with descending neurological s/s such as double vision after eating canned food. ▪ Traveler's diarrhea: E coli is the most common pathogen responsible for traveler's diarrhea. o Peptic Ulcer Disease: associated w/ H. pylori infection. TX: amoxicillin, clarithromycin, and omeprazole for 2 weeks. • Primary care basics: o Data collection: subjective o Documentation: OLDCART best reflects HPI o APN business essentials: ▪ Determining the level of complexity of a visit: risk data and dx ▪ CPT codes: recognized universally and can be used to track healthcare data ▪ EDICARE PART A: covers hospital services only. ▪ MEDICARE PART B: covers provider and outpatient services. ▪ Third party payers: the term indemnity insurer refers to an insurance that pays for the medical care of the insured but does not provide that care. NR511 MIDTERM BEST EXAM PRACTICE REVIEW STUDY GUIDE LATEST UPDATE 2022/2023 NR511 MIDTERM BEST EXAM PRACTICE REVIEW STUDY GUIDE LATEST UPDATE 2022/2023 • 105 lb. • 110 lb. • 120 lb (To estimate a client’s ideal weight, use the following formula: For women older than age 25, allow 100 lb for the first 5 ft, then add 5 lb for each inch thereafter. For men, allow 106 lb for the first 5 ft, then add 6 lb for each inch thereafter. Multiply the number by 110% for a client with a large frame and 90% for a client with a small frame) • 130 lb. 12. Susie, age 5, comes to the clinic for a well-child visit. She has not been in since she was 2. Her immunizations are up to date. What immunizations would you give her today? • None; wait until she is 6 years old to give her her booster shots (Susie is due for diphtheria, tetanus, and pertussis (DTaP); inactivated polio vaccine (IPV); and measles, mumps, and rubella (MMR) between the ages of 4 and 6. There is no need to wait until she is 6 years old) • Diphtheria, tetanus, and pertussis (DTaP); Haemophilus influenzae type B (Hib); and measles, mumps, and rubella (MMR) ( Susie is due for inactivated polio vaccine (IPV), in addition to DTaP and MMR. She is not due for Hib) • Diphtheria, tetanus, and pertussis (DTaP) and inactivated polio vaccine (IPV) (Susie is due for measles, mumps, and rubella (MMR), in addition to DTaP and IPV) • Diphtheria, tetanus, and pertussis (DTaP); inactivated polio vaccine (IPV); and measles, mumps, and rubella (MMR) (Because Susie has not been in for several years, one cannot assume she will come in next year to get the immunizations that are due between the ages of 4 and 6; therefore, this opportunity to give her needed immunizations cannot be missed. Between the ages of 4 and 6, a child is due for DTaP, IPV, and MMR, if all other immunizations are up to date) 13. Mimi, age 52, asks why she should perform a monthly breast self-examination (BSE) when she has her mammograms on schedule. You respond: • “If you are faithful about your annual exams and mammograms, that is enough.” (All women older than age 20 should examine their breasts monthly) • “More breast abnormalities are picked up by mammograms than by clinical exams or BSE.” ( More than 90% of all breast abnormalities are first detected by self-examination_ • “More than 90% of all breast abnormalities are first detected by self-examination.” (More than 90% of all breast abnormalities are first detected by self-examination. All women older than age 20 should examine their breasts monthly, a week after their period. After menopause, women should examine their breasts at the same time each month. There is some controversy. Most women perform BSE incorrectly, thus making the significance of a positive finding questionable. Breastcancer.org recommends a BSE be performed as a mammogram may miss a tumor) • “Self-examinations need to be performed only every other month.” ( All women older than age 20 should examine their breasts monthly) 14. Marian’s husband, Stu, age 72, has temporal arteritis. She tells you that his physician wants to perform a biopsy of the temporal artery. She asks if there is a less invasive diagnostic test. What test do you tell her is less invasive? • Computed tomography (CT) scan (A CT scan and magnetic resonance imaging (MRI) are done to detect neurological damage from hemorrhage, tumor, cyst, edema, or myocardial infarction. These tests may also identify displacement of the brain structures by expanding lesions. However, not all lesions can be detected by NR511 MIDTERM BEST EXAM PRACTICE REVIEW STUDY GUIDE LATEST UPDATE 2022/2023 NR511 MIDTERM BEST EXAM PRACTICE REVIEW STUDY GUIDE LATEST UPDATE 2022/2023 CT scan or MRI) • Magnetic resonance imaging (MRI) (A computed tomography (CT) scan and MRI are done to detect neurological damage from hemorrhage, tumor, cyst, edema, or myocardial infarction. These tests may also identify displacement of the brain structures by expanding lesions. However, not all lesions can be detected by CT scan or MRI) • Electroencephalogram (EEG) (An EEG is used to evaluate the electrical activity of the brain. It can identify seizure activity as well as certain infectious and metabolic conditions) • Color duplex ultrasonography (A biopsy of the temporal artery is usually required to confirm the diagnosis of temporal arteritis. However, color duplex ultrasonography (a combination of ultrasonography and the flow velocity determinations of a Doppler system) has been shown to examine even small vessels, such as the superficial temporal artery, and show a halo around the inflamed arteries when temporal arteritis is present. Therefore, it is a much less invasive procedure than biopsy) 15. Which of the following refers to an aspect of a patient’s health that can be changed or affected by a health intervention? • Modifiable risk factor (A modifiable risk factor is an aspect of a patient’s health that can be changed or affected by a health intervention) • Nonmodifiable risk factor (A nonmodifiable risk factor is something in a patient’s health profile that cannot be changed) • Adjustable risk factor (This term does not exist in medicine) • Changeable risk factor (This term does not exist in medicine) 16. An 81-year-old patient presents for a physical. She recently had a fall and now has problems walking up her stairs. The only restroom in the house is on the second floor. She also has a flight of stairs outside her house she has to navigate in order to reach street level, and this is difficult for her. Where does this information belong in your chart note? • Functional health patterns (The patient is having trouble with her normal routine and daily life due to her recent fall, so this information belongs here) • Review of systems (This section is for patient symptoms, not functional home circumstances) • Plan (While you can address these problems in the plan, this is not the place to make note of them) • Assessment (You can list her immobility as a diagnosis, but this area of the note would not be the place to note these findings) NR511 MIDTERM BEST EXAM PRACTICE REVIEW STUDY GUIDE LATEST UPDATE 2022/2023 NR511 MIDTERM BEST EXAM PRACTICE REVIEW STUDY GUIDE LATEST UPDATE 2022/2023 17. Telehealth has shown a drastic increase in utilization by patients in which of the following fields of medicine? • Psychology and psychiatry. • Pediatrics. • Primary care. • Dermatology. 18. Mark, a 56-year-old man, comes to your practice seeking help quitting smoking. You prescribe varenicline (Chantix), a prescription medication, to aid with his attempt. What instructions do you give Mark regarding how to stop smoking with Chantix? • Start the Chantix today according to the dosing schedule and then quit smoking after the 12-week medication schedule (Following the 12-week medication schedule after quitting minimizes the effect of nicotine withdrawal because Chantix works on the same receptors as nicotine) • Start the Chantix today according to the dosing schedule and then pick a date to stop smoking about 7 days after starting Chantix (Chantix has been shown to be more effective in helping smokers quit than Zyban, another nonnicotine prescription medicine for smoking cessation. While Chantix contains no nicotine, it works on the same receptors as nicotine. It’s the addiction to the nicotine inhaled from smoking that makes quitting so hard. The recommended dosing schedule for Chantix is as follows: day 1 to day 3, one 0.5-mg tablet per day; day 4 to day 7, one 0.5-mg tablet twice a day (once in the morning and once in the evening); day 8 to the end of treatment, one 1-mg tablet twice per day (once in the morning and once in the evening). Chantix should be taken with a full glass of water after eating. The client should choose a quit date to stop smoking. Chantix should be taken for 7 days before the quit date. This lets Chantix build up in the body. Smoking should cease on the quit day, and Chantix should be continued for up to 12 weeks. If the client has not completely quit smoking by 12 weeks, another 12 weeks may help the client stay cigarette-free. The most common side effect is nausea (30%), but this is usually not severe enough to make the client discontinue the medication) • Pick a date to stop smoking and start Chantix that day according to the dosing schedule (Taking Chantix for 7 days before the quit date allows it to build up in the body) • Start Chantix today, take it twice a day for 2 weeks, and then stop smoking (Chantix should be initiated with one tab once a day. It should be taken for 7 days prior to the quit date) 19. Joseph, a 55-year-old man with diabetes, is at your office for his diabetes follow-up. On examining his feet with monofilament, you discover that he has developed decreased sensation in both feet. There are no open areas or signs of infection on his feet. What health teaching should Joseph receive today regarding the care of his feet? • “Wash your feet with cold water only.” (Joseph should wash his feet with warm, soapy water) • “See a podiatrist every two years, inspect your own feet monthly, and apply lotion to your feet daily.” (Diabetic clients should see a podiatrist yearly) • “Go to a spa and have a pedicure monthly.” (Joseph should rely on licensed health professionals for the care of his feet) • “See a podiatrist yearly; wash your feet daily with warm, soapy water and towel dry between the toes; inspect your feet daily for any lesions; and apply lotion to any dry areas.” (The American Diabetes Association recommends careful inspection of a diabetic client’s feet for corns, calluses, and open lesions to prevent further deterioration into diabetic foot ulcers. Joseph should wash his feet daily with warm, soapy water and then towel dry them, especially between the toes, to prevent fungal infections. Diabetic clients should see a podiatrist yearly. Encourage patients to use a mirror to inspect the bottom of their feet) 20. Marvin is a gay man who is ready to “come out.” What is the last step in the process of coming out? • Testing and exploration (Testing and exploration is the second of 4 steps) • Identity acceptance (Identity acceptance is the third of 4 steps) NR511 MIDTERM BEST EXAM PRACTICE REVIEW STUDY GUIDE LATEST UPDATE 2022/2023 NR511 MIDTERM BEST EXAM PRACTICE REVIEW STUDY GUIDE LATEST UPDATE 2022/2023 measurement. When do women lose the greatest amount of bone density? • During adolescence (Bone density increases during adolescence until peak bone density occurs (about age 20). • The first year of menopause (Women lose the greatest amount of bone density in the first decade after menopause) • The first 10 years after menopause (Bone loss begins at a rate of 0.5% a year in a woman’s middle to late 40s. The rate then increases up to 7% a year for the first decade after menopause. This increase in the rate of bone loss is directly related to a decrease in a woman’s estrogen. Bone loss then decreases to 0.5% to 1% a year until death. This patient should be encouraged to do weight-bearing exercises and have an adequate calcium intake of 1000 to 1500 mg/d, with sufficient amounts of vitamin D) • Bone loss occurs continuously at the same rate from menopause to death (Bone loss occurs at different rates from menopause until death) 30. If a screening test used on 100 individuals known to be free of breast cancer identified 80 individuals who did not have breast cancer while missing 20 of the individuals, the specificity would be: • 80% (Specificity measures a screening test’s ability to recognize individuals who are nondiseased or those with negative reactions (true negatives). It can be represented by a ratio of tested negatives to the total number of known, or true, negatives. In this case, the number of negatives that the test recognized was 80, with the total number of known, or true, negatives being 100. Therefore, 80 out of 100 (80/100) equals a specificity of 80%) • 60%. • 40%. • 20%. 31. Julia, age 18, asks you how many calories of fat she is eating when 1 serving has 3 g of fat. You tell her: • 12 cal. • 18 cal. • 27 cal (A gram of fat contains 9 cal, whereas a gram of either carbohydrates or proteins contains 4 cal. In this case, Julia was eating 3 g, or 27 cal, of fat (9 cal per gram of fat times 3 g of fat equals 27 cal). • 30 cal. 32. In relation to writing a patient encounter note, the acronym SOAP stands for which of the following words? • Subjective, objective, assessment, plan. • Symptoms, objective, assessment, pills. • Subjective, outward findings, assessment, plan. • Symptoms, objective, assessment, plan. 33. Which of the following demonstrates a subjective finding? • Pain level. • Pulse rate. • Eye color. • Extremity edema. 34. Which of the following statements about health promotion is false? NR511 MIDTERM BEST EXAM PRACTICE REVIEW STUDY GUIDE LATEST UPDATE 2022/2023 NR511 MIDTERM BEST EXAM PRACTICE REVIEW STUDY GUIDE LATEST UPDATE 2022/2023 • Health promotion is a benefit to add to your practice if you have time (Health promotion is an important aspect of clinical practice and not used solely if it is convenient for the practitioner) • Health promotion helps to prevent diseases (This is included in the scope of health promotion) • Health promotion includes early screening to detect diseases (This is included in the scope of health promotion) • Health promotion includes helping to restore health after a patient has had an illness (This is included in the scope of health promotion) 35. What intervention would not be included in the definition of secondary prevention? • Sexually transmitted infection (STI) testing • Preventive mammography. • Testicular self-examination. • Routine immunizations (This is primary) 34. As a nurse practitioner, which of the following would not be an example of primary health promotion? • Posting health articles on your social media account. • Speaking to patients at each visit about strategies for maintaining/attaining a healthy life. • Lecturing at local community centers about diabetes. • Organizing a 5K running event for the patients in your practice. 35. Which of the following patients is health literate? • A 62-year-old female who speaks the language of her provider, has Medicare for insurance, and can drive (This patient has access to transportation and health insurance and has the ability to communicate with her provider) • A 70-year-old patient with dementia (This patient cannot make his or her own medical decisions) • A 22-year-old migrant worker who only speaks Spanish (This patient may have difficulty understanding a provider in the United States and therefore may not be able to heed medical advice) • A 45-year-old college graduate with no health insurance (This person can be seen only at an emergency department or if they have the ability to pay; their ability to obtain medical care is not guaranteed) 36. The study of the way diseases are spread through groups and what causes and helps spread these diseases is called? • Epidemiology (Epidemiology is the study of the way diseases are spread through groups and what causes and helps spread these diseases) • Pathology (Pathology is the study of diseased tissues and cells) • Physiology (Physiology is the study of the physiological processes of the body) • Kinesiology (Kinesiology is the study of body movement) 37. Documentation in your patient’s chart is important for all the following reasons except? • Documentation is only important for recording your side of the visit so you can reduce your liability in the event of a lawsuit (While an accurate record will help to document your office visit and could serve as a legal document in a lawsuit, it is not the only reason documentation is important) • Documentation allows you to communicate your findings to another provider. • Documentation allows you to remember the office visit so you can best treat the patient in the future. NR511 MIDTERM BEST EXAM PRACTICE REVIEW STUDY GUIDE LATEST UPDATE 2022/2023 NR511 MIDTERM BEST EXAM PRACTICE REVIEW STUDY GUIDE LATEST UPDATE 2022/2023 • Documentation allows you to create a record of the visit and record your patient’s findings and treatment plans. 38. How often should you receive a Tdap booster when no injury history is present? • Every 10 years (You need this vaccine every 10 years. If you suffer an injury between 5 and 10 years after receiving your last vaccine, some providers will go ahead and give you a booster) • After 18 you don’t need a booster unless you have a laceration. • Every 20 years. • Every 5 years. 39. How do you respond when Jill, age 42, asks you how long she should work out each week? • Exercise for at least 30 minutes every day. • Exercise a total of 2 hours per week. • Exercise for at least 20 minutes 3 or more days per week. • Exercise for at least 30 minutes 5 days per week ( The American College of Sports Medicine (ACSM) recommends 3 to 5 days per week for most cardiovascular exercise programs. For cardiovascular benefits, aim for 20 to 60 minutes in your target heart rate zone, apart from the warm-up and cool-down periods. ACSM’s overall recommendation is for most adults to engage in at least 150 minutes of moderate-intensity exercise each week. If you encourage patients to exercise most days of the week, they might make this goal) 40. Sandra, a 27-year-old nurse, states that she does not want to get the hepatitis B virus vaccine because of its adverse effects. You tell her that the most common adverse effect is: • Fatigue (This mild, transient systemic adverse effect occurs in 11% to 17% of adults and 8% to 18% of children) • Headache (This mild, transient systemic adverse effect occurs in 11% to 17% of adults and 8% to 18% of children) • Pain at the injection site (The most common adverse reaction to the hepatitis B virus vaccine is pain at the injection site (13%–20% in adults, 3%–9% in children). • Elevated temperature (This mild, transient systemic adverse effect occurs in 1% to 6% of all injections) 41. You are sharing with your client the idea that he needs to get some counseling to deal with his severe stress because it is affecting his physiological condition. Which of the following hormonal changes occurs during severe stress? • A decrease in catecholamines (During severe stress, as glucagon release increases, catecholamine levels increase. The insulin to glucagon ratio decreases, glycogen breakdown increases, and glucose production from amino acids increases) • An increase in cortisol (During severe stress, the cortisol level increases, allowing mobilization of free fatty acids. This triggers a series of reactions. With the increased cortisol, glucose production from amino acids increases) • A decrease in antidiuretic hormone (The release of antidiuretic hormone also increases during stressful periods, increasing the retention of water) • A decrease in aldosterone (The level of aldosterone increases, leading to the increased retention of sodium) 42. Emily, a healthy 26-year-old woman, asks you how she can prevent bone loss as she ages. She is concerned because both her maternal grandmother and now her mother have severe osteoporosis. What guidance would you give to Emily? • Drink all the soda you like—it has no effect on your bone density (Soda drinking has been shown to decrease bone mass) NR511 MIDTERM BEST EXAM PRACTICE REVIEW STUDY GUIDE LATEST UPDATE 2022/2023 NR511 MIDTERM BEST EXAM PRACTICE REVIEW STUDY GUIDE LATEST UPDATE 2022/2023 occur rapidly or slowly over the course of several years, knowing what stage a family member is in helps family members in planning and knowing what to expect. In stage 3, personality change is marked and depression may occur. Directions must be specific and repeated for safety, recent memory is poor, disorientation occurs easily, people are incorrectly identified, and the person may be lethargic) • Stage 4 (In stage 4, apathy is noticeable, memory is poor or absent, urinary incontinence is present, individuals are not recognized, and the person should not be alone) 52. When performing a sports physical exam on Kevin, a healthy 16-year-old boy, which question in the history is important to ask Kevin or his guardian? • Did anyone in your family ever have sudden cardiac death? (The risk of sudden death during sports activities from hypertrophic cardiomyopathy may be greatly reduced with a thorough cardiac history and examination. If a child has a relative who died of sudden cardiac disease before age 55, that child could possibly have hypertrophic cardiomyopathy) • Does anyone in your family have elevated cholesterol levels? • Did you ever have any injury requiring stitches? • Does anyone in your family have a history of asthma? 53. The Agency for Healthcare Research and Quality (AHRQ) was established to: • Mandate treatment protocols • Dictate health care policy based on voluminous research. • Promote evidence-based practice (The AHRQ was established to promote evidence-based practice and develop databases for research and clinical guidelines. The AHRQ routinely publishes reviews of studies on clinical problems with summaries of treatment protocols and effectiveness) • Develop cost-effective interventions. 54. You have seen a client who has tested positive for syphilis. You have treated the client; tested the client for other potential sexually transmitted diseases, including human immunodeficiency virus (HIV); counseled the client about safe sexual practices; and scheduled the client to return at 3 and 6 months for repeat serologic testing. The tests at those times demonstrated that no further syphilis was present. Should you have taken any other action? • No, you have treated the client appropriately (You must report the case to the local health authorities) • Yes, you must report the case to the local health authorities (The practitioner is also responsible for reporting the case of syphilis to the local health authorities. Syphilis is easily treated and controllable if its presence is reported) • Yes, you need to notify all sexual contacts (All sexual partners of the client should be contacted; however, it is the health department that has the trained staff who will investigate contacts and follow up) • Yes, you must follow up on the client’s HIV status (It is not necessary to retest the client’s HIV status unless there is a new clinical reason on subsequent visits) 55. Which of the following is the best method for evaluating the efficacy of a new clinical intervention? • A case report. • A descriptive study. • A randomized controlled trial (The best method for evaluating the efficacy of a new clinical intervention is a randomized controlled trial (RCT). Case reports, descriptive studies, and correlational studies are methodological approaches that are less reliable in establishing causal relationships, and thus the attribution of an effect to the new clinical intervention would be less clear. The effect might be attributable to other confounding variables) NR511 MIDTERM BEST EXAM PRACTICE REVIEW STUDY GUIDE LATEST UPDATE 2022/2023 NR511 MIDTERM BEST EXAM PRACTICE REVIEW STUDY GUIDE LATEST UPDATE 2022/2023 • A correlational study. 56. Negligence is the predominant legal theory of malpractice liability. Negligence includes: • Failure to give necessary care (Failure to give necessary care, failure to follow up, failure to refer when necessary, and failure to disclose necessary information to patients are all examples of negligence) • Failure to discharge. • Failure to share information with family. • Failure to provide shelter and food. 57. While counseling Mr. Brown, a patient newly diagnosed with diabetes, the advanced practice registered nurse (APRN) hands him a pamphlet on foot care. She notices that he puts it away, saying he will read it later. What might this behavior indicate? • Being overly dependent on others. • Having a low literacy level (The National Assessment of Adult Literacy reported that 1 out of every 3 Americans lacks the literacy needed to understand health care providers and is embarrassed to admit it. One sign of lack of literacy is putting aside written information) • Having a high level of motivation. • Memorization of the information that is being taught. 58. If conflict arises during a job negotiation, you should consider: • Separating the issue from the person (Always separate the issue from the person. It is about achieving a mutually satisfying outcome—a win-win situation. Clarifying misconceptions and focusing on what has been achieved thus far can be an effective strategy. You want to leave the process with positive feelings even if a work agreement is not achieved) • Always standing your ground. • Getting a legal opinion. • Basing your actions on the contract a friend of yours has secured in a local practice. 59. There are advantages to owning your own practice. However, there are also barriers to the ability to do this. These barriers include which of the following? • Getting and keeping a collaborating physician, if required by law; getting on managed care panels; and getting privileges at hospitals ( Barriers to independent practice include getting on managed care panels; getting and keeping a collaborating physician, if mandated in the state where one is practicing; getting referrals from hospital emergency rooms; getting privileges at hospitals; and lacking legal authority in the state of practice to admit to nursing homes, to order home care services, and/or to direct hospice services) • Getting privileges at hospitals, getting referrals from hospital emergency departments, and lack of knowledge. • Inability to find a collaborating physician, inability to find clients, and lack of empathy. NR511 MIDTERM BEST EXAM PRACTICE REVIEW STUDY GUIDE LATEST UPDATE 2022/2023 NR511 MIDTERM BEST EXAM PRACTICE REVIEW STUDY GUIDE LATEST UPDATE 2022/2023 • Lack of legal authority to admit clients to nursing homes, to order home care, and/or to direct hospice services, and lack of ability to manage client care without clear medical oversight and supervision. 60. Which of the following statements related to statistical techniques and their usage in research is true? • Statistical significance and clinical significance are the same. • If a journal article you are writing is required to be limited in length, you should delete the descriptive statistics and keep the inferential statistics (If a journal article you are writing is required to be limited in length, you should not delete the descriptive statistics and keep the inferential statistics. You must describe your sample and possibly a number of other things before explaining your inferences) • Correlational coefficients imply causality (Correlational coefficients measure the strength of the relationship between variables; correlation does not imply causality, nor can you make firm predictions based on these data) • To determine the appropriate statistical test to use, consider sample size, level of measurement, and data type (This is true; to determine the appropriate statistical test to use, consider sample size, level of measurement, and data type, as well as other factors) 61. Prescriptive authority for advanced practice registered nurses (APRNs): • Is permitted only under protocol. • Is mandated by law in more than 40 states. • Varies by state (Prescriptive authority for APRNs varies by state) • Includes the ability to prescribe controlled substances. 62. Mr. Griffin, age 85, has been given a diagnosis of bowel cancer, and surgery is indicated. He is mentally alert; however, he is refusing to give consent for the procedure. You respond by: • Ordering a psychiatric consultation. • Having your collaborating physician talk with the client. • Respecting his wishes (Respecting the client’s wishes is the most correct response to this situation. You may want to consider a psychological overlay, such as depression, and be sure this is not a driving component of the client’s behavior. However, if the client appears to be of sound mind, his wishes should be respected) • Talking with his family. 63. When caring for a client who speaks a language different from yours, the ideal strategy is to: • Use gestures to convey the meaning of words and use a foreign language dictionary of medical terms. • Rely on family members to interpret ( may be necessary to rely on family members to translate, but it is not the best strategy. For example, it is often the school-age child who has the best grasp of English; however, relying on the child to interpret reverses parent-child roles and places an unnecessary and sometimes inappropriate burden on the child. It also lessens the client’s sense of authority and privacy) • Review the case first with an interpreter before beginning the clinical visit (If you are in a setting where an interpreter is available, reviewing the case with them before seeing the client is the ideal strategy for enhancing cross-cultural communication. Information about the reason for the visit and the purpose of the health care encounter can be exchanged beforehand, potentially enhancing communication) • Use a pad and pencil to pass information back and forth with an interpreter. 64. The term collaboration is best defined as: • Interdisciplinary teamwork (Collaboration is essential in interdisciplinary teamwork, but NR511 MIDTERM BEST EXAM PRACTICE REVIEW STUDY GUIDE LATEST UPDATE 2022/2023 NR511 MIDTERM BEST EXAM PRACTICE REVIEW STUDY GUIDE LATEST UPDATE 2022/2023 • Asking several quick and specific questions in rapid succession to establish the exact nature of this emergent clinical situation. • Asking open-ended questions to elicit pertinent clinical data (Asking open-ended questions is essential for establishing a therapeutic relationship, even in an emergency situation, and it is a good interviewing technique) • Reassuring the client that he is in good hands, in a well-equipped emergency department, and that everything will be okay. • Asking the client about his anxiety level. 71. Certain characteristics differentiate research from quality improvement. Which of the following is an example of research? • Client satisfaction being evaluated relative to existing practice. • An intervention—well supported in the literature—being implemented and evaluated. • A standardized assessment tool (eg, risk assessment for falls) being implemented and evaluated. • A new intervention being implemented and compared with current practice to determine which is better (One characteristic that differentiates research from quality improvement is its comparison of new interventions with current practices to determine which is better. Research asks new questions that generate new knowledge when answered with a degree of generalizability. Additionally, there may be an implied risk to a human subject —ie, clients receiving the new and untested interventions may be at risk) 72. Which of the following statements about Medicaid is true? • Medicaid is a federal plan created to provide care for indigent persons. • Medicaid pays for family planning services, dental care, and eyeglasses. • Eligibility requirements for Medicaid are mandated by the Health Care Financing Administration. • Medicaid is a program for the indigent financed jointly by the federal and state governments (Financed jointly by the federal and state governments, Medicaid is a program created to pay for health care services for the indigent. Minimally, Medicaid must provide inpatient, skilled nursing facility, and home care; physician services; outpatient care; family planning services; and periodic screening, detection, and treatment of children under age 12) 73. Advanced practice registered nurses (APRNs) are affected by laws and rules, although these vary by state. Which of the following is affected by state laws and regulations? • Delegation of authority by physicians (APRNs must abide by laws and regulations, such as those related to scope of practice, reimbursement for health care services, delegation of authority by physicians, quality of care, and requirements for collaboration) • How many clients you must see every hour. • Universal health care law. • Making no more than 5 referrals for 1 client. 74. What must you do as an advanced practice registered nurse (APRN) before billing for visits? • Establish a collaborative agreement with a physician. • Obtain a provider number and familiarize yourself with the rules and policies of the third-party payer (To bill your clients for services, you must obtain a provider number or panel membership (as needed) and familiarize yourself with the rules and policies of each payer) • Provide evidence of continuing medical education. • Obtain a Drug Enforcement Administration (DEA) number. 75. Because of the potential for exploitation of older adults, the geriatric population is designated a vulnerable one NR511 MIDTERM BEST EXAM PRACTICE REVIEW STUDY GUIDE LATEST UPDATE 2022/2023 NR511 MIDTERM BEST EXAM PRACTICE REVIEW STUDY GUIDE LATEST UPDATE 2022/2023 when it comes to obtaining informed consent for serving as a research subject. Safeguards you should adhere to when conducting research on a geriatric population include which of the following? • Assessing the competence of the individual before obtaining consent (Safeguards to follow when conducting research on a geriatric population include assessing the competence of the individual before obtaining consent, making sure the consent form is in understandable language, and obtaining verbal consent (in some situations, verbal consent is all that is required) • Obtaining permission from the family or staff • Stressing how important the research is and why their participation and perspective, as older adults, are important. • Determining if the research is exempt, in which case you do not need to obtain consent to participate. NR511 MIDTERM BEST EXAM PRACTICE REVIEW STUDY GUIDE LATEST UPDATE 2022/2023 NR511 MIDTERM BEST EXAM PRACTICE REVIEW STUDY GUIDE LATEST UPDATE 2022/2023 76. Relapse is a common phenomenon seen during behavioral changes. Useful strategies for the health care provider to institute to aid the client in a relapse situation include: • Using fear to reinforce the need to change. • Telling the client that the relapse is a learning opportunity in preparation for the next action stage (Useful strategies for the health care provider to institute to aid the client in a relapse situation include telling the client that the relapse is a learning opportunity in preparation for the next action stage (ie, positive reframing). • Stressing the need to stay “straight.” • Involving the family in stressing the need to stay “straight.” 77. Mrs. Smith, age 85, lost her husband 6 months ago. Since that time, she has been overwhelmed and has had difficulty coping. Today, she is in your office, tearful, weak, and discouraged. Her mobility is also becoming increasingly limited because of her need for a hip replacement. She is indecisive, expresses fear about the surgery, and tells you she does not want the surgery. Your best action is to: • Treat the client’s psychological problems and provide support (Your best action is to treat the client’s psychological problems and provide support. With Mrs. Smith, there are enough symptoms to warrant evaluation and possible intervention for psychological problems—most likely depression) • Respect the client’s wishes. • Consider placing the client in an assisted living facility. • Explain to the client that she is depressed and will feel better after the surgery. 78. An emancipated minor is a client who is younger than age 18 but who is considered a competent adult with the authority to accept or refuse medical treatment. How do you determine if the 16-year-old you are seeing is an “emancipated minor”? • No 16-year-old would be considered an emancipated minor. • Although definitions vary among states, the term usually implies that the minor has entered into a valid marriage, is a member of the military, or has been granted this status by a court (To treat this client as an emancipated minor, you need some proof of marriage, active military status, or papers from a court attesting to the minor’s status as emancipated. To petition a court for emancipation, a minor must be a certain age (ie, 14-16, depending on the state law), not live with his or her parents or guardian, be capable of managing financial affairs, and have the ability to provide for his or her well-being) • The client claims that he is free from all parental control. • The client is accompanied by an older friend who states that he or she is the client’s guardian and will accept legal responsibility. 79. An effective method for assessing a client’s retention and understanding of educational materials is: • Asking the client to restate what you have reviewed (Methods used to assess your client’s retention and understanding of educational materials include asking your client to restate or do a return demonstration of what you have reviewed, having your client keep a diary or record of his or her behaviors, and reviewing written materials with your client) • Providing a pathophysiology book for your client to take home and read. • Repeating your explanations of disease pathophysiology. • Objective testing. 80. When counseling on health behavior change, the advanced practice registered nurse (APRN) would say which of the following? NR511 MIDTERM BEST EXAM PRACTICE REVIEW STUDY GUIDE LATEST UPDATE 2022/2023 NR511 MIDTERM BEST EXAM PRACTICE REVIEW STUDY GUIDE LATEST UPDATE 2022/2023 new treatments on clients; and disseminating those results in clinical practice) • Organizing and conducting a research study. • Data collection. • Ensuring protection of human subjects. 86. Your client is convinced her illness has been caused by the ill will of a family member. Her description of her illness is an example of her: • Explanatory reasoning (Explanatory reasoning—reasoning that explains, in the client’s view, the cause of his or her illness—is a term attributed to anthropologist Arthur Kleinman, who developed the explanatory model) • Lack of understanding of scientific medicine. • Delusional ideation. NR511 MIDTERM BEST EXAM PRACTICE REVIEW STUDY GUIDE LATEST UPDATE 2022/2023 NR511 MIDTERM BEST EXAM PRACTICE REVIEW STUDY GUIDE LATEST UPDATE 2022/2023 • Cultural bias. 87. As an advanced practice registered nurse (APRN), you begin a new job in a small practice with 2 other physicians. You have been hired to be a partner in the practice, but the eldest partner is reluctant to allow you to take on new clients. This is an example of: • Role stress. • Role strain. • Role insufficiency. • Role conflict (Role conflict can result from perceived differences in the expected role of a nurse practitioner (NP) joining a practice. The NP can often be perceived as competition, and some may view him or her as an extender of medical care and not as an independent practitioner) 88. Certain characteristics differentiate research from quality improvement. Which of the following is an example of research? . • Client satisfaction being evaluated relative to existing practice. • An intervention—well supported in the literature—being implemented and evaluated. • A standardized assessment tool (eg, risk assessment for falls) being implemented and evaluated. • A new intervention being implemented and compared with current practice to determine which is better (One characteristic that differentiates research from quality improvement is its comparison of new interventions with current practices to determine which is better. Research asks new questions that generate new knowledge when answered with a degree of generalizability. Additionally, there may be an implied risk to a human subject —ie, clients receiving the new and untested interventions may be at risk) 89. The primary purpose of an institutional review board (IRB) is to: • Protect human subjects (The purpose of an IRB is to protect human subjects, as its primary function is ethical. It does not evaluate the scientific merit of proposed research, nor does it oversee and coordinate the research efforts of an institution or an individual. These functions are usually performed by the institution’s research committee. It is your obligation as a researcher to obtain some form of IRB approval any time you are conducting research on human subjects. If you are planning a research project in a private practice setting, approval should be obtained from the IRB of an affiliated institution, such as a hospital where you are credentialed and permitted to admit clients) • Evaluate the scientific merit of proposed research • Oversee and coordinate the research efforts of an institution. • Oversee and coordinate the research efforts of an individual. 90. The term indemnity insurer refers to an insurer: • In a health maintenance organization (HMO). • In a preferred provider organization (PPO). • That pays for the medical care of the insured (The term indemnity insurer refers to an insurer that pays for the medical care of the insured but does not provide that care) • That pays health care providers on a per-visit, per-procedure basis. 91. Most health maintenance organizations (HMOs) use a reimbursement mechanism called capitation. This means that the: NR511 MIDTERM BEST EXAM PRACTICE REVIEW STUDY GUIDE LATEST UPDATE 2022/2023 NR511 MIDTERM BEST EXAM PRACTICE REVIEW STUDY GUIDE LATEST UPDATE 2022/2023 • HMO reimburses the provider on a fee-for-service basis • HMO reimburses the provider a predetermined fee per client per month based on the client’s age and sex (The reimbursement mechanism called capitation that some HMOs use is one in which the HMO reimburses the provider a set fee per client per month based on the client’s age and sex. HMOs are prepaid, comprehensive systems of health benefits that combine both financing and delivery of services to subscribers. They may pay providers on a capitated or fee-for-service basis) • Fee paid to the provider fluctuates with the treatment. • Provider is reimbursed by each individual client or family. 92. Mrs. Smith, age 85, lost her husband 6 months ago. Since that time, she has been overwhelmed and has had difficulty coping. Today, she is in your office, tearful, weak, and discouraged. Her mobility is also becoming increasingly limited because of her need for a hip replacement. She is indecisive, expresses fear about the surgery, and tells you she does not want the surgery. Your best action is to: NR511 MIDTERM BEST EXAM PRACTICE REVIEW STUDY GUIDE LATEST UPDATE 2022/2023 NR511 MIDTERM BEST EXAM PRACTICE REVIEW STUDY GUIDE LATEST UPDATE 2022/2023 had), a standard of care must have been breached (which Sally did when she did not check his potassium level when she increased the dosage of diuretic), and harm or damage must have occurred as a result of the duty and the breach of the standard of care (which did not occur). Because all of the components were not met, malpractice has not been established) • No, because she took remedial action. • Yes, because she was negligent. 99. You are working in an emergency department as an advanced practice registered nurse (APRN). An adolescent boy is brought in, unconscious, with a head injury after being struck by a car. He has no identification, and there is no parent or adult with him. What should you do? • Provide the appropriate medical treatment even if it involves surgery ( The emergency treatment exception allows you to treat minors in emergency or life-threatening situations when a parent or guardian cannot be reached to give consent for treatment. This includes transfusions when necessary. The legal definition of an emergency medical condition is any condition that threatens the loss, impairment, or serious dysfunction of life or limb or causes severe pain) • Do everything except order a blood transfusion, even if it is indicated, because you do not know the client’s religious preferences. • Call the hospital attorney before instituting any care. • Contact all local police stations in an attempt to identify the client and find his parents before instituting treatment. 100. You are attempting to elicit a history from Mr. Barnes during his first visit to your office. He is becoming increasingly angry and belligerent. He says, “Can’t you hurry up? Dr. Smith never takes this long! Why are all these questions necessary?” You respond: • “I’m sorry, Mr. Barnes, but I need these questions answered.” • “I want to provide the best possible care for you, Mr. Barnes.” • “Perhaps your wife can assist with some of these questions.” • “You seem very upset, Mr. Barnes. Could you share with me what is bothering you?” (The most important aspect of communicating with clients is acknowledging their feelings. By responding, “You seem very upset, Mr. Barnes. Could you share with me what is bothering you?” you are acknowledging Mr. Barnes’s discomfort by reflecting back his feelings, and you are offering to assist him) 101. Mr. Brill, age 50, is a house painter who has smoked 2 to 3 packs of cigarettes per day since he was 20 years old. He comes in to the clinic complaining of a chronic cough. When you discuss his smoking behavior, he states, “I know I need to stop smoking, but I’m under too much stress right now.” Mr. Brill is at which stage of change? • The precontemplative stage. • The contemplative stage (Mr. Brill’s response indicates that he is in the contemplative stage of change. He is considering change but has not taken any action. You would focus your teaching on reinforcing his understanding of the need to change, teaching him the skills needed to make the change, pointing out the positive aspects of making the change, and stressing his ability to do so) • The action stage. • The maintenance stage. 102. Which of the following strategies can help foster client compliance? • In-depth client education. • High-frequency medication dosing. • Providing positive feedback and reinforcement (Strategies that help prevent client noncompliance include NR511 MIDTERM BEST EXAM PRACTICE REVIEW STUDY GUIDE LATEST UPDATE 2022/2023 NR511 MIDTERM BEST EXAM PRACTICE REVIEW STUDY GUIDE LATEST UPDATE 2022/2023 providing positive feedback and reinforcement and performing careful follow-up on canceled or missed appointments) • Monitoring serum drug levels to assess therapeutic range. 103. Which of the following nonverbal communication techniques is important in the establishment of rapport with a client? • Taking notes only while the client is talking. • Making direct eye contact with the client, with periodic breaks to take or check notes (Nonverbal communication techniques important in the establishment of rapport with a client include making direct eye contact, with periodic breaks to take or check notes; avoiding having a desk between you and the client; and having personal grooming appropriate to the setting. Other nonverbal communication techniques that help NR511 MIDTERM BEST EXAM PRACTICE REVIEW STUDY GUIDE LATEST UPDATE 2022/2023 NR511 MIDTERM BEST EXAM PRACTICE REVIEW STUDY GUIDE LATEST UPDATE 2022/2023 establish rapport include sitting while interviewing the client rather than standing, sitting near the client but not invading their personal comfort zone, and maintaining a friendly, helpful expression) • Having a desk between you and the client. • Wearing jeans in the clinical setting to ensure your comfort. 104. Denial of provider status is something that seriously impedes a nurse practitioner’s ability to practice. If that occurs, some steps one can take include: • Requesting that your clients lobby on your behalf, going to the newspapers, and reapplying. • Requesting that your physician colleagues intervene on your behalf and writing critical letters to the organization in question. • “Bashing” the organization to others, reapplying, and contacting an attorney. • Writing letters to the organization’s president and chief executive officer (CEO), activating others to lobby on your behalf, and reapplying after a 6-month period (There are many steps a nurse practitioner can take if denied provider status by a third-party reimburser. First, one should ascertain the reasons for this stance and determine whether it is the same across the board regarding nurse practitioners. If it is a consistent policy, attempt to find out why and begin marshaling evidence to overturn this stance in a constructive way. This may include having both clients and physician colleagues “lobby” on your behalf. Find out who the decision maker in the organization is and attempt to communicate directly with that person. Ascertain if there is a law in the state mandating this policy. Be prepared to testify at hearings and speak out at community meetings about this issue. Request language changes that specify “ask your doctor” and lobby to have these changes adopted. Reapplication in 6 months is reasonable) 105. When teaching your client about medication you are prescribing, the most important point(s) to discuss initially is(are): • The action of the drug and its adverse effects. • Whether to take the drug on a full or empty stomach. • What it is for, how much to take, and when to take it (It can be difficult to find time to do client teaching; therefore, apply the rule of 3 Ss: short, specific, and simple. The most important points you need to teach your client initially about a new drug are what the medication is for, how much to take, and when to take it) • What to do if the client experiences any adverse effects. 106. Which of the following statements about case management is true? • Case management oversees the client throughout acute care hospitalization. • Case management is organized around a system of interdisciplinary resources and services (Case management is organized around a system of interdisciplinary resources and services; the clinical and financial aspects of care are overseen by a case manager who has a financial incentive to manage risk and maximize the quality of care) • Case management depends on physician-driven leadership to oversee the illness episode. • Case management is applicable to the rehabilitative portion of the illness episode. 107. Elder abuse and neglect are increasing concerns, and it is estimated that 4% to 10% of older Americans are abused or neglected. What is the legal responsibility of the health care provider in reporting elder abuse and neglect? • The health care provider should discuss the suspected abuse or neglect with the client. • The health care provider should discuss the suspected abuse or neglect with the client’s family. • The health care provider must report the suspected abuse or neglect to the appropriate state protective agency (If a health care provider suspects elder or dependent-adult abuse or neglect, the provider, in most states, NR511 MIDTERM BEST EXAM PRACTICE REVIEW STUDY GUIDE LATEST UPDATE 2022/2023 NR511 MIDTERM BEST EXAM PRACTICE REVIEW STUDY GUIDE LATEST UPDATE 2022/2023 Week 2 Practice Questions 1. Jill, a 34-year-old bank teller, presents with symptoms of hay fever. She complains of nasal congestion, runny nose with clear mucus, and itchy nose and eyes. On physical assessment, you observe that she has pale nasal turbinates. What is your diagnosis? • Allergic rhinitis (The symptoms of hay fever, also called allergic rhinitis, are similar to those of viral rhinitis but usually persist and are seasonal in nature. When assessing the nasal mucosa, you will observe that the turbinates are usually pale or violaceous because of venous engorgement) • Viral rhinitis (with viral rhinitis, the turbinates are typically erythematous) • Nasal polyps (With nasal polyps, there are usually yellowish, boggy masses of hypertrophic mucosa) • Nasal vestibulitis from folliculitis (Nasal vestibulitis usually results from folliculitis of the hairs that line the nares) 2. A 75-year-old African American male presents to your family practice office complaining of visual impairment. He has worn corrective lenses for many years but has noticed that his vision has gotten progressively worse the past 6 months. He denies pain. He states his vision is worse in both eyes in the peripheral aspects of his visual field. He also notes trouble driving at night and halos around street lights at night. You test his intraocular pressure, and it is 23 mm Hg. What is his most likely diagnosis? • Open-angle glaucoma (This is the typical presentation of chronic, or open-angle, glaucoma) • Angle-closure glaucoma (This is an acute, painful form of glaucoma) • Cataracts (This is a loss of central vision) • Macular degeneration (This also affects central vision) 3. Which of the following is not a cause of conductive hearing loss? • Presbycusis (This is a cause of sensorineural hearing loss) • Cerumen impaction. • Otitis media. • Otosclerosis. 4. Which of the following is not a complication of untreated group A streptococcal pharyngitis? • Glomerulonephritis. • Rheumatic heart disease. • Scarlet fever. • Hemolytic anemia (This is a complication of mononucleosis) 5. Alexandra, age 34, was treated with oral antibiotics 2 weeks ago for a urinary tract infection. She is seen in the office today for a follow-up visit. On physical examination, the nurse practitioner notices that she has some painless, white, slightly raised patches in her mouth. This is probably caused by: • Herpes simplex (Herpes simplex (a viral infection) sores are usually discrete and not spread over a large area) • Aphthous ulcers (Aphthous ulcers (canker sores) are extremely painful) • Candidiasis (Painless, white, slightly raised patches in a client’s mouth are probably caused by candidiasis (thrush) • Oral cancer (Cancerous lesions are usually discrete and not spread over a large area) 6. Mattie, age 64, presents with blurred vision in 1 eye and states that it felt like “a curtain came down over my eye.” She doesn’t have any pain or redness. What do you suspect? • Retinal detachment (The classic sign of retinal detachment is a client stating that “a curtain came down over my eye.” Typically, the person presents with blurred vision in 1 eye that becomes progressively worse, with no pain or redness) NR511 MIDTERM BEST EXAM PRACTICE REVIEW STUDY GUIDE LATEST UPDATE 2022/2023 NR511 MIDTERM BEST EXAM PRACTICE REVIEW STUDY GUIDE LATEST UPDATE 2022/2023 • Acute angle-closure glaucoma (In older adults with acute angle-closure glaucoma, there is a rapid onset, with severe pain and profound visual loss. The eye is red, with a steamy cornea and a dilated pupil) • Open-angle glaucoma (In older adults with open-angle glaucoma, there is an insidious onset, a gradual loss of peripheral vision over a period of years, and a perception of “halos” around lights) • Cataract (With a cataract, there is blurred vision that is progressive over months or years and no pain or redness) NR511 MIDTERM BEST EXAM PRACTICE REVIEW STUDY GUIDE LATEST UPDATE 2022/2023 NR511 MIDTERM BEST EXAM PRACTICE REVIEW STUDY GUIDE LATEST UPDATE 2022/2023 7. While doing a face, head, and neck examination on a 16-year-old patient, you note that the palpebral fissures are abnormally narrow. What are you examining? • The nasolabial folds (The nasolabial folds are the skin creases that extend from the angle of the nose to the corners of the mouth) • The openings between the margins of the upper and lower eyelids (The palpebral fissures are the openings between the margins of the upper and lower eyelids. Someone who appears to be squinting is said to have narrow palpebral fissures) • The thyroid gland in relation to the trachea (The thyroid is a butterfly-shaped gland located in the front of the neck, just below the Adam’s apple; it is wrapped around the trachea) • The distance between the trigeminal nerve branches (The trigeminal nerve is the fifth cranial nerve located within the brain. It is composed of 3 branches—ophthalmic, maxillary, and mandibular—and is primarily responsible for transmitting sensations from the face to the brain. It is also the nerve that controls the muscles used for chewing) 8. When the Weber test is performed with a tuning fork to assess hearing and there is no lateralization, the nurse practitioner should document this finding as: • Conductive deafness (With conductive deafness, sound lateralizes to the defective ear because it is transmitted through bone rather than air) • Perceptive deafness (With perceptive deafness, sound lateralizes to the better ear) • A normal finding (A Weber test assesses hearing by bone conduction. With normal hearing, sound is heard equally well in both ears, meaning there is no lateralization) • Nerve damage (Damage to cranial nerve VIII (CN VIII), the vestibulocochlear nerve, causes symptoms of hearing loss, vertigo, and loss of equilibrium) 9. What significant finding(s) in a 3-year-old child with otitis media with effusion would prompt more aggressive treatment and referral? • There is a change in the child’s hearing threshold to greater than 25 dB (If a child with otitis media with effusion has a change in the hearing threshold greater than 25 dB and has notable speech and language delays, more aggressive treatment is indicated. When the child’s hearing examination reveals a change in the hearing threshold, it is extremely important that the provider evaluate the child’s achievement of developmental milestones in speech and language. Any abnormal findings warrant referral) • The child has become a fussy eater. • The child’s speech and language skills seem slightly delayed. • Persistent rhinitis is present. 10. A 25-year-old client who plays in a band complains that he finds it difficult to understand his fellow musicians at the end of a night of performing, a problem that is compounded by the noisy environment of the club. These symptoms are most characteristic of which of the following? • Sensorineural loss (Sensorineural loss comes from exposure to loud noises, inner ear infections, tumors, congenital and familial disorders, and aging. The results of the Weber and Rinne tests will assist in the diagnosis) • Conductive loss (The etiology of conductive loss includes ear infection, presence of a foreign body, perforated drum, and otosclerosis of the ossicles) • Tinnitus (Tinnitus is ringing in the ears. The client does not complain of this symptom) • Vertigo (Vertigo is dizziness associated with inner ear dysfunction. The client does not complain of this symptom) 11. A 64-year-old obese woman comes in complaining of difficulty swallowing for the past 3 weeks. She states that “some foods get stuck” and she has been having “heartburn” at night when she lies down, especially if she has had a heavy meal. Occasionally, she awakes at night coughing. She denies weight gain and/or weight loss, vomiting, or change in bowel movements or color of stools. She denies alcohol and tobacco use. There is no pertinent family history or findings on review of systems (ROS). Physical examination is normal, with no abdominal tenderness, and the stool is occult blood (OB) negative. What is the most likely diagnosis? NR511 MIDTERM BEST EXAM PRACTICE REVIEW STUDY GUIDE LATEST UPDATE 2022/2023 NR511 MIDTERM BEST EXAM PRACTICE REVIEW STUDY GUIDE LATEST UPDATE 2022/2023 have no pathological significance) • Pingueculae (Pingueculae are yellowish, elevated nodules appearing on the sclera. They are caused by a thickening of the bulbar conjunctiva from prolonged exposure to the sun, wind, and dust) • The result of arcus senilis (Arcus senilis appears as gray-white arcs or circles around the limbus and is a result of deposits of lipid material that make the cornea look cloudy) • Actinic keratosis (Actinic keratoses are wartlike growths on the skin that occur in middle-aged or older adults and are caused by excessive exposure to the sun) NR511 MIDTERM BEST EXAM PRACTICE REVIEW STUDY GUIDE LATEST UPDATE 2022/2023 NR511 MIDTERM BEST EXAM PRACTICE REVIEW STUDY GUIDE LATEST UPDATE 2022/2023 17. Cynthia, a 31-year-old woman with a history of depression, is seen in the office today for complaints of headaches. She was recently promoted at her job, and this has caused increased stress. She describes the headache as a tightening (viselike) feeling in the temporal and nuchal areas. The pain is bilateral and tends to wax and wane. It started approximately 2 days ago and is still present. What kind of headache is she describing? • Classic migraine (Classic migraine headaches occur after an aura, which may include visual, auditory, or olfactory symptoms) • Tension headache (Tension headache is the most common type of headache. These headaches are usually bilateral and tend to wax and wane but have a characteristic pressure/tightening (viselike grip) in the forehead, temporal, or nuchal areas and can last from minutes to weeks. Tension headache may be associated with stress, depression, or anxiety disorders) • Sinus headache (Sinus headache would usually be precipitated by allergies or cold symptoms. These headaches are described as pain, pressure, or fullness in the cheeks, brow, or forehead, with worsening pain when leaning forward) • Cluster headache (Cluster headaches come in clusters, with exquisite pain that awakens the client from sleep. They are more common in males than females) 18. In a young child, unilateral purulent rhinitis is most often caused by: • A foreign body (In a young child, unilateral purulent rhinitis is most often caused by a foreign body. The key word here is unilateral) • A viral infection (Viral infections usually affect both nares) • A bacterial infection (Bacterial infections usually affect both nares) • An allergic reaction (Allergic reactions usually affect both nares) 19. Kevin, age 26, has AIDS and presents to the clinic with complaints of a painful tongue covered with what look like creamy white, curdlike patches overlying erythematous mucosa. You are able to scrape off these “curds” with a tongue depressor, which assists you in making which of the following diagnoses? • Leukoplakia (Leukoplakia cannot be removed by rubbing the mucosal surface; it appears as little white lesions on the tongue) • Lichen planus (Oral lichen planus is a chronic inflammatory autoimmune disease; it also has white lesions that do not rub off) • Oral candidiasis (Oral candidiasis (thrush) is distinctive because the white areas on the tongue can be rubbed off with a tongue depressor. Thrush may be seen in denture wearers, in debilitated clients, and in those who are immunocompromised or taking corticosteroids or broad-spectrum antibiotics) • Oral cancer (Oral cancer must be ruled out in any lesion because early detection is the key to successful management and a good prognosis) 20. You diagnose 46-year-old Mabel with viral conjunctivitis. Your treatment should include: • Gentamicin ophthalmic ointment (Antibiotics should not be used in clients with viral conjunctivitis) • Ciprofloxacin ophthalmic drops (Antibiotics should not be used in clients with viral conjunctivitis) • Supportive measures and lubricating drops (artificial tears) (Viral conjunctivitis is treated with supportive measures, including cold compresses and lubricating eye drops. Preventive measures, such as frequent handwashing, are important, as viral conjunctivitis is highly contagious) • Oral erythromycin for 14 days (Antibiotics should not be used in clients with viral conjunctivitis) 21. The antibiotic of choice for recurrent acute otitis media (AOM) and/or treatment failure in children is: • Amoxicillin (Amoxil) (Amoxicillin (Amoxil) is used as the first-line treatment of AOM. However, it is not used in patients with recurrent AOM or treatment failure) • Amoxicillin and potassium clavulanate (Augmentin) (The antibiotic of choice for recurrent AOM or treatment failure is amoxicillin and potassium clavulanate (Augmentin) • Azithromycin (Zithromax) (Azithromycin (Zithromax) for otitis media is usually reserved for patients with penicillin allergy) • Prednisone (Deltasone) (Prednisone (Deltasone) is not an antibiotic) 22. Kevin, a 56-year-old lawyer, has throbbing pain in the left eye, an irregular pupil shape, marked photophobia, NR511 MIDTERM BEST EXAM PRACTICE REVIEW STUDY GUIDE LATEST UPDATE 2022/2023 NR511 MIDTERM BEST EXAM PRACTICE REVIEW STUDY GUIDE LATEST UPDATE 2022/2023 and redness around the iris. What is your initial diagnosis? • Conjunctivitis (A client with conjunctivitis has redness more prominently at the periphery of the eye, along with tearing and itching. The client may also complain of a scratchy, burning, or gritty sensation but not pain, although photophobia may be present) NR511 MIDTERM BEST EXAM PRACTICE REVIEW STUDY GUIDE LATEST UPDATE 2022/2023 NR511 MIDTERM BEST EXAM PRACTICE REVIEW STUDY GUIDE LATEST UPDATE 2022/2023 27. A 75-year-old female presents to your office complaining of dizziness and hearing loss. The patient states she awoke yesterday with dizziness, which she described as feeling the room spinning. She also notes intermittent ringing in her ears. On physical exam, the patient has lateralization of her hearing loss to the unaffected ear. Rinne test shows air conduction lasts longer than bone conduction. What is the next step in helping this patient’s symptoms? • Order a computed tomography (CT) scan to rule out acoustic neuroma. • Start her on high-dose Augmentin. • Start the patient on a low-salt, low-caffeine diet and give her meclizine for vertigo attacks (This is the treatment for symptoms of Meniere disease. You would also want to consider an ear, nose, and throat consult to rule out other etiologies of the patient’s symptoms. Meniere disease is diagnosed based on history and the exclusion of other conditions as well) • Immediate referral to an ear, nose, and throat (ENT) specialist. 28. A 6-year-old female presents to your pediatric office with her mother complaining of right ear pain for 3 days. This pain resolved with Tylenol. The patient has also had noted fevers of 101.3°F over the last 2 nights. The patient had a nonproductive cough for 7 days prior to the ear pain. On physical exam, the patient has tenderness with tugging on the auricle of the ear. The tympanic membrane is not mobile with pneumatic otoscopy and is erythematous and full. The patient has no drainage from the ear and no mastoid tenderness. What is the next step? • Symptom management and reassurance that symptoms will resolve with time. • Computed tomography (CT) of the head. • Amoxicillin 80 to 90 mg/kg/d (This is the treatment of choice for acute otitis media) • Augmentin 45 mg/kg/d (This is the treatment of choice for chronic/treatment-resistant otitis media) 29. You prescribe Levaquin (levofloxacin) for a severe sinus infection. What is not a possible adverse reaction to this medication? • Achilles tendon rupture. • Peripheral neuropathy. • Nephrotoxicity. • Stevens-Johnson syndrome (Stevens-Johnson syndrome is usually caused by allopurinol and beta- lactam antibiotics) 30. A 20-year-old male presents to your primary care clinic. This patient is a college student. He complains of fatigue, sore throat, and low-grade fever for 3 days. On physical exam, he has a temperature of 100.7°F. His ear exam is normal. His nose and throat exam shows mild erythema of the nasal mucosa and edematous, enlarged tonsils bilaterally, with erythema of the pharyngeal wall and tonsillar exudates. He has inflamed posterior cervical lymph nodes. He has a mild nonproductive cough and clear lung exam. What is his most likely diagnosis? • Viral pharyngitis. • Mononucleosis (This presentation could be a viral pharyngitis; however, with posterior cervical lymphadenitis, you would suspect mononucleosis) • Streptococcal pharyngitis. • Upper respiratory infection. 31. Which of the following is not recommended for hoarseness? • Vocal rest. • Tobacco cessation. • Decrease in caffeine use. • Oral steroids (Oral steroids are not routinely used to treat hoarseness) 32. Marlene, a 57-year-old cashier, comes to the clinic because she is unable to differentiate between sharp and dull stimulation on both sides of her face. You suspect: NR511 MIDTERM BEST EXAM PRACTICE REVIEW STUDY GUIDE LATEST UPDATE 2022/2023 NR511 MIDTERM BEST EXAM PRACTICE REVIEW STUDY GUIDE LATEST UPDATE 2022/2023 • Bell palsy (Bell palsy affects the facial nerve, resulting in weakness or paralysis of 1 side of the face) • A lesion affecting the trigeminal nerve (A lesion affecting the sensory portion of the trigeminal nerve could be manifested by bilateral symptoms) • A stroke—brain attack, cerebrovascular accident (CVA) (A stroke is unilateral in its presentation) • Shingles (Shingles is unilateral in its presentation) 33. Sharon, a 47-year-old bank teller, is seen by the nurse practitioner in the office for a red eye. You are trying to decide between a diagnosis of conjunctivitis and iritis. One distinguishing characteristic between the two is: • Eye discomfort (Clients with iritis and those with conjunctivitis both complain of eye discomfort, although in iritis the pain is moderately severe, with intermittent stabbing) NR511 MIDTERM BEST EXAM PRACTICE REVIEW STUDY GUIDE LATEST UPDATE 2022/2023 NR511 MIDTERM BEST EXAM PRACTICE REVIEW STUDY GUIDE LATEST UPDATE 2022/2023 • Slow progression (Both conditions generally produce a slowly progressive redness.) • A ciliary flush (When trying to decide between a diagnosis of conjunctivitis and iritis, one distinguishing characteristic is the ciliary flush present in iritis. Photophobia is not usually present in conjunctivitis, but it is always present with iritis. Photophobia occurs with corneal inflammation, iritis, and angle-closure glaucoma) • No change in or slightly blurred vision (Vision is normal with conjunctivitis and blurred with iritis) 34. Susan is a 19-year-old college student and avid swimmer. She frequently gets swimmer’s ear and asks if there is anything she can do to help prevent it other than wearing earplugs, which do not really work for her. What do you suggest? • Start using a cotton-tipped applicator to dry the ears after swimming. • Use ear drops made of a solution of equal parts alcohol and vinegar in each ear after swimming (Using ear drops made of a solution of equal parts alcohol and vinegar in each ear after swimming is effective in drying the ear canal and maintaining an acidic environment, therefore preventing a favorable medium for the growth of bacteria, the cause of swimmer’s ear) • Use a hair dryer on the highest setting to dry the ears. • Stop swimming 35. Which of the following conditions produces sharp, piercing facial pain that lasts for seconds to minutes? • Trigeminal neuralgia (Trigeminal neuralgia is described as a sharp, piercing, shooting facial pain that is severe but usually lasts only a short time. The origin is cranial nerve V (CN V), the trigeminal nerve) • Temporomandibular joint (TMJ) syndrome (TMJ dysfunction is associated with pain on opening and closing the mouth and is also associated with crepitus of that joint) • Goiter (A goiter is generally painless) • Preauricular adenitis (Preauricular adenitis (enlarged and inflamed preauricular nodes) would be sustained until the etiological cause was identified and treated) 36. Mary, age 82, presents with several eye problems. She states that her eyes are always dry and look “sunken in.” What do you suspect? • Hypothyroidism (With hyperthyroidism, the eyes appear to bulge out (exophthalmos), but in hypothyroidism, the eyes do not appear any different) • Normal age-related changes (Dryness of the eyes and the appearance of “sunken” eyes are normal age- related changes) • Cushing syndrome (A moon face is apparent with Cushing syndrome, and this might make the eyes appear to be sunken in, although on close examination, they are not) • A detached retina (With a detached retina, the outward appearance is normal, but the client complains of seeing floaters or spots in the visual field and describes the sensation as like a curtain being drawn across the vision) 37. Marjorie, age 37, has asthma and has been told she has nasal polyps. What do you tell her about them? • Nasal polyps are usually precancerous. • Nasal polyps are benign growths (Nasal polyps are benign growths that occur frequently in clients with sinus problems, asthma, and allergic rhinitis. Polyps are neither neoplastic growths nor precancerous, but they do have the potential to affect the flow of air through the nasal passages. Clients who have asthma and have nasal polyps may have an associated allergy to aspirin, a syndrome that is referred to as Samter triad.) • The majority of nasal polyps are neoplastic. • They are probably inflamed turbinates, not polyps, because polyps are infrequent in clients with asthma. 38. Erica, age 39, has a sudden onset of shivering, sweating, headache, aching in the orbits, and general malaise and misery. Her temperature is 102°F. The nurse practitioner diagnosed her with influenza (flu). What is your next course of action? NR511 MIDTERM BEST EXAM PRACTICE REVIEW STUDY GUIDE LATEST UPDATE 2022/2023 NR511 MIDTERM BEST EXAM PRACTICE REVIEW STUDY GUIDE LATEST UPDATE 2022/2023 • Acute rhinosinusitis (The client is exhibiting classic characteristics of acute rhinosinusitis) • Chronic rhinosinusitis. • Nasal tumor 44. Which of the following statements about macular degeneration is not true? • Macular degeneration is characterized by gradual loss of peripheral vision (This is how open- angle glaucoma is characterized. Macular degeneration is gradual loss of central vision) • Macular degeneration is the leading cause of blindness in people younger than 60. • Tobacco use is a risk factor for macular degeneration. • There are 2 different types of macular degeneration: wet and dry. NR511 MIDTERM BEST EXAM PRACTICE REVIEW STUDY GUIDE LATEST UPDATE 2022/2023 NR511 MIDTERM BEST EXAM PRACTICE REVIEW STUDY GUIDE LATEST UPDATE 2022/2023 45. What is the most common bacterial pathogen associated with acute otitis media? • Streptococcus pneumonia (This causes 40% to 50% of cases) • Haemophilus influenza (This causes 10% to 30% of cases) • Streptococcus pyogenes (this is an uncommon cause) • Moraxella (Branhamella) catarrhalis (This is an uncommon cause) 46. You are the nurse practitioner caring for Martha, a 47-year-old accountant. You have made a diagnosis of acute sinusitis based on Martha’s history and the fact that she complains of pain behind her eye. Which sinuses are affected? • Maxillary (Maxillary sinus pain is felt over the cheek and into the upper teeth) • Ethmoid (With ethmoid sinus problems, the pain is felt behind the eye and high on the nose) • Frontal (Frontal sinus pain is felt over the lower forehead_ • Sphenoid (Sphenoid sinus pain is felt in the occiput, vertex, or middle of the head) 47. Jonathan, age 19, has just been given a diagnosis of mononucleosis. Which of the following statements is true? • The offending organism is a bacterium, and Jonathan should be treated with antibiotics (Antibiotic therapy is not indicated for Epstein-Barr virus (EBV) • Convalescence is usually only a few days, and Jonathan should be back to normal in a week (The patient should be instructed to avoid stress and that convalescence may take several weeks) • Mononucleosis is rarely contagious (The virus that causes mononucleosis is transmitted through saliva, hence the nickname the “kissing disease.” It is contagious and can be transmitted through kissing or sharing utensils. Bed rest is necessary only in severe cases) • Jonathan should avoid contact sports and heavy lifting (When teaching clients about mononucleosis, or Epstein-Barr virus (EBV), tell them to avoid contact sports and heavy lifting because of splenomegaly and a threat of rupture) 48. A 65-year-old man presents complaining of a left-sided, deep, throbbing headache and mild fatigue. On examination, the client has a tender, tortuous temporal artery. You suspect giant cell arteritis (GCA), or temporal arteritis. What is the least invasive procedure to help with diagnosis? • Magnetic resonance imaging (MRI) of the head (Magnetic resonance angiography (MRA), not MRI, can be done to evaluate the blood vessels in the brain, but the blood test is less invasive) • Erythrocyte sedimentation rate (ESR) (An elevated ESR—anywhere from 30 to 100 mm/h—may be seen in giant cell arteritis (temporal arteritis); however, the ESR may also be normal. The temporal artery supplies the optic nerve; if temporal arteritis is suspected due to the age of the client (50 and older) and the location and character of the pain, it is essential that a referral to a surgeon be made for immediate biopsy of the artery before damage to the optic nerve occurs. A temporal artery biopsy (TAB), an invasive procedure, is the criterion standard for diagnosing temporal arteritis) • Electroencephalogram (EEG) (EEG is not used in the diagnosis of GCA) • Otoscopy (Otoscopy is not used in the diagnosis of GCA.) 49. When you are assessing the internal structure of the eye of your 59-year-old patient, the absence of a red reflex may indicat (When assessing the internal structure of the eye, absence of a red reflex may indicate the total opacity of the pupil because of a cataract or hemorrhage into the vitreous humor. It may also be a result of improper positioning of the ophthalmoscope) • A cataract or hemorrhage into the vitreous humor (When assessing the internal structure of the eye, absence of a red reflex may indicate the total opacity of the pupil because of a cataract or hemorrhage into the vitreous humor. It may also be a result of improper positioning of the ophthalmoscope) • Acute iritis (Acute iritis is noted by constriction of the pupil accompanied by pain and circumcorneal redness (ciliary flush) • Nothing; this is a normal finding in older adults. • Diabetes or long-standing hypertension (If areas of hemorrhage, exudate, and white patches are present when the internal structure of the eye is assessed, they are usually a result of diabetes or long- NR511 MIDTERM BEST EXAM PRACTICE REVIEW STUDY GUIDE LATEST UPDATE 2022/2023 NR511 MIDTERM BEST EXAM PRACTICE REVIEW STUDY GUIDE LATEST UPDATE 2022/2023 standing hypertension) 50. Mia, a 27-year-old school teacher, has a 2-day history of severe left ear pain that began after 1 week of upper respiratory infection (URI) symptoms. On physical examination, you find that she has acute otitis media (AOM). She has a severe allergy to penicillin. The most appropriate antimicrobial option for this patient is: • Ciprofloxacin (Cipro) (Ciprofloxacin, a fluoroquinolone antibiotic, is not recommended for the treatment of acute otitis media) NR511 MIDTERM BEST EXAM PRACTICE REVIEW STUDY GUIDE LATEST UPDATE 2022/2023 NR511 MIDTERM BEST EXAM PRACTICE REVIEW STUDY GUIDE LATEST UPDATE 2022/2023 55. A 25-year-old male presents with “bleeding in my eye” for 1 day. He awoke this morning with a dark area of redness in his eye. He has no visual loss or changes. He denies constitutional symptoms, pruritus, drainage, or recent trauma. The redness presents on physical exam as a dark red area in the patient’s sclera of the right eye only and takes up less than 50% of the eye. The patient’s remaining sclera is clear and white. He also notes he was drinking alcohol last night and vomited afterward. What is the best treatment? • Topical steroids and close follow-up with an ophthalmologist (his would be the treatment for an infectious process; however, topical steroids need to be used in the eye with care) • Sending the patient to the emergency department for immediate ophthalmology consult (This would be the treatment for a globe rupture or acute angle-closure glaucoma) • Reassurance that this lesion will resolve without any treatment in 2 to 4 weeks (This is the classic presentation of a subconjunctival hemorrhage. It will resolve without treatment in 2 to 4 weeks. Vomiting probably caused his hemorrhage) • Cold compresses and frequent handwashing (This is the treatment for viral conjunctivitis) 56. What is the most common cause of epistaxis? • Digital trauma (Picking the nose is the most common cause of nosebleeds) • Warfarin. • Vitamin C deficiency (This can cause nasal tissue friability and epistaxis) • Hemophilia A (This would increase the risk of bleeding from the nose, but hemophilia itself is not a cause of epistaxis) 57. Aaron, age 4, is brought in to the clinic by his father. His tympanic membrane is perforated from otitis media. His father asks about repair of the eardrum. How do you respond? • “The eardrum, in most cases, heals within several weeks.” (Most perforated tympanic membranes seen with acute otitis media heal within several weeks) • “We need to schedule Aaron for a surgical repair.” (If the eardrum has not healed within 3 to 6 months, a surgical repair can be done, but not until age 7 to 9 years) • “He must absolutely stay out of the water for 3 to 6 months.” (Aaron can swim on the surface with the use of an ear mold but must not dive, jump, or swim underwater) • “If the eardrum is not healed in several months, it can be surgically repaired.” (If the eardrum has not healed within 3 to 6 months, a surgical repair can be done, but not until age 7 to 9 years) 58. Martin, age 24, presents to the office with an erythematous ear canal and pain on manipulation of the auricle. He is on vacation and has been swimming daily at the resort. What is your diagnosis? • Acute otitis media (acute otitis media is painful, is usually a result of cotton swab use or physical trauma, and usually follows an upper respiratory infection) • Chronic otitis media (Chronic otitis media is usually not painful, although the ear may be painful during an exacerbation) • External otitis (With external otitis, there is pain, an erythematous ear canal, and usually a history of recent swimming) • Temporomandibular joint (TMJ) syndrome (Ear pain may also be the result of TMJ dysfunction. It is usually made worse by chewing or grinding the teeth) 59. A 44-year-old banker comes to your office for evaluation of a pulsating headache over the left temporal region, and he rates the pain as an 8 on a scale of 1 to 10. The pain has been constant for the past several hours and is accompanied by nausea and sensitivity to light. He has had frequent, though less severe, headaches for many years, and they are usually relieved by over-the-counter medicines. He is unclear as to a precipitating event but notes that he has had visual disturbances before each headache and he has been under a lot of stress in his job. Based on this description, what is the most likely diagnosis of this type of headache? • Tension (Tension headaches are not associated with photophobia and are usually bilateral and associated with limited neck range of motion (ROM). Tension headaches are not preceded by an aura) • Migraine (Migraines are classically preceded by an aura and accompanied by nausea, vomiting (sometimes), and photophobia. They are usually unilateral) • Cluster (Cluster headaches are unilateral, frequently occur at night, and bear some resemblance to NR511 MIDTERM BEST EXAM PRACTICE REVIEW STUDY GUIDE LATEST UPDATE 2022/2023 NR511 MIDTERM BEST EXAM PRACTICE REVIEW STUDY GUIDE LATEST UPDATE 2022/2023 migraines; however, cluster headaches are accompanied by tearing, nasal stuffiness, and sweating on the same side as the headache, and they come in clusters.) • Temporal arteritis (The client is not in the age range (older than 50) for temporal arteritis) NR511 MIDTERM BEST EXAM PRACTICE REVIEW STUDY GUIDE LATEST UPDATE 2022/2023 NR511 MIDTERM BEST EXAM PRACTICE REVIEW STUDY GUIDE LATEST UPDATE 2022/2023 60. Max, age 35, states that he thinks he has an ear infection because he just flew back from a business trip and feels unusual pressure in his ear. You diagnose barotrauma. What is your next action? • Prescribe nasal steroids and oral decongestants (Barotrauma of the auditory canal, causing a sensation of abnormal middle ear pressure, may be relieved by the use of nasal steroids and oral decongestants) • Prescribe antibiotic ear drops (With barotrauma, there is no infection, just swelling of the airways, which causes the sensation of abnormal pressure; therefore, antibiotics are not indicated) • Prescribe systemic antibiotics (With barotrauma, there is no infection, just swelling of the airways, which causes the sensation of abnormal pressure; therefore, antibiotics are not indicated) • Refer Max to an ear, nose, and throat specialist (This condition is certainly within the practitioner’s scope of practice, and a referral is not indicated) 61. Your client, a 72-year-old smoker of 50 years, is at the office today for a routine physical. During your inspection of the oral mucosa, you discover a white lesion on the lateral surface of the tongue that you suspect to be cancerous. You document your finding as: • A superficial, translucent, subepithelial, vesicle-like lesion in the oral mucosa (A superficial mucocele presents as a small, clear vesicle on noninflamed mucosa) • A white, painless, firm, ulcerated lesion with indurated borders (Approximately 90% of oral cancers are squamous cell carcinoma (SCC), which is seen typically on the lip or lateral part of the tongue, usually as a lesion that is white, red, or mixed white and red. SCCs are characterized by painless, firm lesions with indurated borders) • An abnormal white coating of the dorsal surface of the tongue (The term “hairy tongue” is used to describe an abnormal coating on the top (dorsal) surface of the tongue. It is a relatively common, temporary, and harmless condition that occurs in as much as 13% of the population. Hairy tongue can occur at any age but is more frequent in older age) • A round, smooth, firm lump on the lateral side of the tongue (A round, smooth, firm lump attached at its base or by a stalk to part of the oral cavity tissue is called a fibroma. It may occur anywhere in the oral cavity but most often develops on the inner lining of the cheeks and lips. A fibroma is a common benign oral cavity tumor) 62. At the clinic, you are assessing Kyle, a 4-month-old baby, for the first time and notice that both eyes are turning inward. What is this called? • Pseudostrabismus (Pseudostrabismus has the appearance of strabismus because of the presence of epicanthic folds but is normal in young children) • Strabismus (Strabismus, also called tropia, is the constant malalignment of the eye axes. It is likely to cause amblyopia) • Esotropia (Esotropia is the inward turning of the eyes) • Exotropia (Exotropia is the outward turning of the eyes) 63. You diagnose acute epiglottitis in Sally, age 5, and immediately send her to the local emergency room. Which of the following symptoms would indicate that an airway obstruction is imminent? • Reddened face. • Screaming. • Grabbing her throat. • Stridor (In a pediatric client with acute epiglottitis, a number of symptoms can indicate that airway obstruction is imminent: stridor, restlessness, nasal flaring, as well as the use of accessory muscles of respiration) 64. Mrs. Johnson, a 54-year-old accountant, presents to the office with a painful red eye without discharge. You should suspect • Bacterial conjunctivitis (With bacterial conjunctivitis, there is purulent, thick discharge) • Viral conjunctivitis (With viral conjunctivitis, there is usually a watery discharge) NR511 MIDTERM BEST EXAM PRACTICE REVIEW STUDY GUIDE LATEST UPDATE 2022/2023 NR511 MIDTERM BEST EXAM PRACTICE REVIEW STUDY GUIDE LATEST UPDATE 2022/2023 • Needs to be tested again because one reading is not indicative of immunity. • Is permanently immune to hepatitis B (The marker for permanent immunity, hepatitis B surface antibodies in the serum, will be present 4 to 10 months after exposure and immunity to hepatitis B) NR511 MIDTERM BEST EXAM PRACTICE REVIEW STUDY GUIDE LATEST UPDATE 2022/2023 NR511 MIDTERM BEST EXAM PRACTICE REVIEW STUDY GUIDE LATEST UPDATE 2022/2023 • Has an acute hepatitis B infection (Hepatitis B surface antigen is the earliest indicator of the presence of an acute infection and is present 4 to 12 weeks after exposure. This marker is also indicative of a chronic infection) 71. Margie, age 52, has an extremely stressful job and was just given a diagnosis of gastric ulcer. She tells you she is sure it is going to be malignant. How do you respond? • “Don’t worry. Gastric ulcers are not cancerous.” • “About 95% of gastric ulcers are benign.” (About 95% of gastric ulcers are benign even though some of these seem to look malignant on x-ray) • “You have about a 50% chance of having gastric cancer from your ulcer.” • “Even if it is cancer, surgery is 100% successful.” 72. Susan, age 59, has no specific complaints when she comes in for her annual examination. She does, however, have type 2 diabetes mellitus (DM), slight hypertension, dyslipidemia, and central obesity. How would you diagnose her? • As a healthy adult with several problems. • As having a glycemic event (If Susan’s DM were out of control, she would have a “glycemic event.) • As having metabolic syndrome (Susan has a constellation of symptoms known as metabolic syndrome. The World Health Organization (WHO), National Cholesterol Education Program Adult Treatment Panel III (NCEP ATP III), and International Diabetes Federation (IDF) have slightly different criteria for this diagnosis. They all, however, include hypertension, dyslipidemia, and central obesity.) • As having multiple organ dysfunction (Eventually, Susan may end up with multiple organ dysfunction. You might consider that she is headed toward that result) 73. Tom has just been diagnosed with celiac disease. Which of the following might you tell him? • There is a new pharmaceutical cure for celiac disease. • A strict gluten-free diet is the only treatment for celiac disease (A strict gluten-free diet is the only treatment for celiac disease) • Your children will not be at a higher risk for developing this disease (Patients with first- or second- degree relatives affected by celiac disease are at higher risk for developing it) • The presence of celiac disease is decreasing dramatically in the United States (The prevalence of celiac disease in the United States has increased dramatically. Approximately 1% of today’s US residents are affected by it) 74. Matt, age 26, recently returned from a camping trip and has gastroenteritis. He says that he has been eating only canned food. Which of the following pathogens do you suspect • Campylobacter jejuni (C jejuni is found primarily in eggs and poultry but may be found in domestic animals) • Clostridium botulinum (C botulinum is an anaerobic, gram-positive bacillus that produces toxins. It is widely distributed in the soil and vegetation. Improperly processed home-canned low-acid vegetables and contaminated meats are the usual cause of food-borne botulism) • Clostridium perfringens (C perfringens is found in soil, feces, air, and water. Outbreaks are caused most often by contaminated meat) • Staphylococcus (Staphylococcus is a common cause of food poisoning. It is caused by the ingestion of an enterotoxin found in improperly handled or stored foods) 75. Ruby has a colostomy and complains that her stools are too loose. What food(s) do you suggest to help thicken the stools? • Cheese (Cheese, bread, pasta, rice, pretzels, and yogurt all help to thicken stools) • Leafy green vegetables (This may loosen) • Raw fruits and vegetables (This may loosen) • Dried beans (This may loosen) 76. Anson tells you he thinks his antacids are causing his diarrhea. You respond: • “Antacids contain fructose, which may not be totally absorbed, resulting in fluid being drawn into the NR511 MIDTERM BEST EXAM PRACTICE REVIEW STUDY GUIDE LATEST UPDATE 2022/2023 NR511 MIDTERM BEST EXAM PRACTICE REVIEW STUDY GUIDE LATEST UPDATE 2022/2023 bowel.” (Fructose is not usually contained in antacids. Fructose is present in apple juice, pear juice, grapes, honey, dates, nuts, figs, and fruit-flavored soft drinks) • “Antacids contain sorbitol or mannitol, which are sugars that aren’t absorbed and can cause fluid to be drawn into the bowel.” (Sorbitol and mannitol are not usually contained in antacids. Sorbitol or mannitol is present in apple juice, pear juice, sugarless gums, and mints) NR511 MIDTERM BEST EXAM PRACTICE REVIEW STUDY GUIDE LATEST UPDATE 2022/2023 NR511 MIDTERM BEST EXAM PRACTICE REVIEW STUDY GUIDE LATEST UPDATE 2022/2023 • Klebsiella (Klebsiella may result in diarrhea, but because Harvey just came back from Mexico, his traveler’s diarrhea is most likely caused by E coli) • Staphylococci (Staphylococci may result in diarrhea, but because Harvey just came back from Mexico, his traveler’s diarrhea is most likely caused by E coli) 83. Marian, age 52, is obese. She complains of a rapid onset of severe right upper quadrant abdominal cramping pain, nausea, and vomiting. Your differential diagnosis might be: • Appendicitis (Pain associated with appendicitis would typically be near the navel, progressing to the right lower quadrant) • Crohn’s disease (In Crohn’s disease, the pain and cramping in the abdomen are more diffuse and not usually accompanied by nausea and vomiting) • Cholecystitis (A rapid onset of severe right upper quadrant (RUQ) abdominal cramping pain with nausea and vomiting is a classic presentation of acute cholecystitis; 90% to 95% of clients with acute cholecystitis also have gallstones. Other symptoms include low-grade fever, epigastric tenderness, guarding, and pain on inspiration during palpation of the RUQ (Murphy sign). The 7 F’s of cholecystitis are fair, fat, 40, female, fertile, fat intolerant, and flatulent.) • Irritable bowel syndrome (In irritable bowel syndrome (IBS), the pain and cramping in the abdomen are more diffuse and not usually accompanied by nausea and vomiting. The pain with IBS originates over some area of the colon, with the left lower quadrant being most often affected) 84. Which of the following is not true regarding hepatitis C? • The greatest rate of infection in the general population is seen in people born between 1945 and 1965. • Many hepatitis C infections are asymptomatic. • Hepatitis C is seen more frequently in men than women. • If hepatitis is asymptomatic it doesn’t cause cirrhosis or liver cancer (This is untrue; the reason many people end up with cirrhosis or liver cancer is because the disease can often be asymptomatic) 85. A 75-year-old male presents for a routine physical. He is obese and has no abdominal pain or recent injuries or problems. He has no complaints. He lies supine for his abdominal exam. He is nontender to palpation and has a normal exam. When he sits up you see a large mass protrude from his abdomen. It is central to his abdomen and inferior to his rib cage. It disappears when he is sitting up fully. What is the patient’s diagnosis? • Ventral (epigastric) hernia (This is the hernia described in the question; most of these hernias are asymptomatic) • Inguinal hernia (This is in the groin and most commonly palpated via a testicular exam) • Femoral hernia (This is in the groin and most common in women) • Umbilical hernia (This is present at the umbilicus, not where it is described in the question) 86. What is the best diagnostic test to confirm the diagnosis of celiac disease? • Anti-tTG IgA (This is the most sensitive and specific test for diagnosing celiac disease) • Anti-dsDNA (This test is positive in a diagnosis of lupus) • Colonoscopy (This would be the best diagnostic tool to use for irritable bowel disease) • Anti-CCP protein (This is a test to diagnose rheumatoid arthritis) 87. A 50-year-old female presents to the urgent care clinic complaining of left lower quadrant pain. She has associated nausea and vomiting, and her vital signs are as follows: temperature 102.5°F, pulse 110, blood pressure 150/90, pulse oximetry 99% on room air. What is the best test to evaluate this patient? • Sigmoidoscopy (This patient has symptoms of diverticulitis; a sigmoidoscopy would be painful but would help rule out cancer) • Abdominal series (This could be done but is unlikely to diagnose diverticulitis) • Computed tomography (CT) scan with oral contrast (This is the best diagnostic choice. While diverticulitis is likely the cause of the patient’s symptoms, a CT of the abdomen would show if the patient has any gynecologic etiology (such as ovarian cyst or tumor) of this pain) NR511 MIDTERM BEST EXAM PRACTICE REVIEW STUDY GUIDE LATEST UPDATE 2022/2023 NR511 MIDTERM BEST EXAM PRACTICE REVIEW STUDY GUIDE LATEST UPDATE 2022/2023 • Abdominal ultrasound (This would show the patient’s ovaries and uterus but would not show the colon) 88. Which oral medication might be used to treat a client with chronic cholelithiasis who is a poor candidate for surgery? NR511 MIDTERM BEST EXAM PRACTICE REVIEW STUDY GUIDE LATEST UPDATE 2022/2023 NR511 MIDTERM BEST EXAM PRACTICE REVIEW STUDY GUIDE LATEST UPDATE 2022/2023 • Ursodiol (Actigall) (Ursodiol (Actigall) is an oral bile acid that dissolves gallstones. For dissolution, 8 to 10 mg/kg per day is given in 2 to 3 divided doses; for prevention, 300 mg twice per day is given. The safety of its use after 24 months has not been established) • Ibuprofen (Advil) (Nonsteroidal anti-inflammatory drugs (NSAIDs), such as ibuprofen (Advil), may be very irritating to the gastrointestinal mucosa) • Prednisone (Deltasone) (Steroids, such as prednisone (Deltasone), may mask an infection as well as irritate the gastric mucosa) • Surgery is the only answer (Surgery is not the only solution for chronic cholelithiasis) 89. You auscultate Julie’s abdomen and hear a peritoneal friction rub. Which condition do you rule out? • Peritonitis (A peritoneal friction rub, which sounds like a rough, grating sound, occurs over organs with a large surface area in contact with the peritoneum when there is peritoneal inflammation (peritonitis) • A liver or spleen abscess (When a peritoneal friction rub is heard over the lower right rib cage, it may be caused by an abscess or tumor of the liver. When heard over the lower left rib cage in the left anterior axillary line, it may indicate infection of the spleen or an abscess or tumor of the spleen) • A liver or spleen metastatic tumor (When a peritoneal friction rub is heard over the lower right rib cage, it may be caused by an abscess or tumor of the liver. When heard over the lower left rib cage in the left anterior axillary line, it may indicate infection of the spleen or an abscess or tumor of the spleen) • Irritable bowel syndrome (Irritable bowel syndrome does not produce a friction rub) 90. Nausea is difficult to discern in a young child. What question might you ask to determine if a child has nausea? • “Are you sick to your tummy?” (Young children sometimes equate being “sick to my tummy” with vomiting and thus might answer no when questioned about nausea) • “Are you hungry?” (To elicit information concerning nausea in a young child, ask the child about hunger because a young child cannot usually differentiate between hunger and mild nausea) • “Are you eating the way you normally eat?” (A young child might not understand this question) • “Are you nauseous?” (A young child might not understand this question) 91. You are counseling Lillian, who is lactose intolerant, about foods to avoid. You know she misunderstands the teaching when she tells you she can have: • Yogurt (Fermented dairy products, such as aged or hard cheeses and cultured yogurt, are easier to digest and contain less lactose than other dairy products) • Foods containing whey (Advise clients who are lactose intolerant to avoid foods containing whey. Whey is a lactose-rich ingredient found in some foods, so clients who are lactose intolerant need to read labels on all foods. To control symptoms, dietary lactose should be reduced or restricted by using lactose-reduced and lactose-free dairy products or by eating lactose-rich foods in small amounts or in combination with low- lactose or lactose-free foods) • Prehydrolyzed milk (Most stores carry milk that has been pretreated with lactase, making it more than 70% lactose-free. This reduced-lactose milk is also known as prehydrolyzed milk) • Oranges (Lillian can eat oranges) 92. Martina, age 34, has AIDS and currently suffers from diarrhea. You suspect she has which protozoal infection of the bowel? • Giardiasis (Giardiasis is also a protozoal infection affecting the intestine, but because Martina has AIDS, the answer must be cryptosporidiosis) • Amebiasis (Amebiasis is also a protozoal infection affecting the intestine, but because Martina has AIDS, the answer must be cryptosporidiosis) • Cryptosporidiosis (Cryptosporidiosis, a protozoal infection of the bowel, is common in immunocompromised clients. It causes villous atrophy and mild inflammatory changes and may secrete an enterotoxin) • Escherichia coli (E coli is a gram-negative bacterium) NR511 MIDTERM BEST EXAM PRACTICE REVIEW STUDY GUIDE LATEST UPDATE 2022/2023 NR511 MIDTERM BEST EXAM PRACTICE REVIEW STUDY GUIDE LATEST UPDATE 2022/2023 98. Dottie brings in her infant, who has gastroesophageal reflux. What do you tell her about positioning her infant? • “Always position infants on their back to prevent sudden infant death syndrome.” (Babies with gastroesophageal reflux should, from birth, be placed to sleep on their back on a firm, flat mattress that is not elevated. Elevating the sleeping surface for back-sleeping babies does not reduce reflux and is not recommended. In addition, these babies should be fed a formula thickened with rice cereal and held in an upright position for 1 hour after feeding, as gravity helps prevent reflux.) • “Rotate your infant between lying on the back and on the stomach.” (In babies with gastroesophageal reflux, the risk of sudden death when the baby is in the tummy- or side-sleeping position outweighs any benefits of tummy or left side positioning of babies.) NR511 MIDTERM BEST EXAM PRACTICE REVIEW STUDY GUIDE LATEST UPDATE 2022/2023 NR511 MIDTERM BEST EXAM PRACTICE REVIEW STUDY GUIDE LATEST UPDATE 2022/2023 • “Your infant should be placed on the left side.” (In babies with gastroesophageal reflux, the risk of sudden death when the baby is in the tummy- or side-sleeping position outweighs any benefits of tummy or left side positioning of babies) • “Place your infant in whatever position she remains quiet.” (Babies with gastroesophageal reflux should, from birth, be placed to sleep on their back on a firm, flat mattress that is not elevated) 99. A 45-year-old homeless man presents to your urgent care clinic for evaluation. His chief complaint is diarrhea. The patient states he started to have diarrhea 2-3 days ago, and it is getting progressively worse. He also notes nausea without vomiting, dry mouth, and double vision. On exam you notice his pupillary reflex is absent. The patient states he lives on the street and eats mostly canned goods that he scavenges from a grocery store dumpster. What is the likely cause of the patient’s symptoms? • Botulism (This patient has a gastrointestinal illness associated with descending neurological symptoms after eating canned food; this is the presentation of botulism) • Salmonella (This is a bacterial illness not normally associated with neurological changes) • Lyme disease (The patient has no skin eruptions or tick bites) • Vitamin C deficiency (Vitamin C deficiency, or scurvy, is normally associated with bleeding gums and easy bruising) 100. What is the most common bacterial cause of traveler’s diarrhea? • Escherichia coli (E coli is the most common pathogen responsible for traveler’s diarrhea) • Campylobacter jejuni. • Salmonella. • Shigella. 101. A 54-year-old female presents to your primary care office for routine reevaluation for gastroesophageal reflux disease (GERD). She has been treated with diet modifications and 6 weeks of omeprazole without improvement of her symptoms. What is the next step in management of this patient’s GERD? • Order an endoscopy (This is the next step in treatment in order to evaluate the etiology of the patient’s GERD and consider biopsy if necessary) • Order a Helicobacter pylori blood test (The next step in care is an endoscopy. If warranted, a biopsy can be done and sent for H pylori at that time) • Try adding ranitidine to the patient’s regimen (H2 antagonists are considered a less aggressive treatment for GERD and would likely not help the patient’s symptoms) • Try adding bismuth to the patient’s regimen (Bismuth can be added to help treat Helicobacter pylori, but that diagnosis has not yet been made) 102. A 25-year-old male presents complaining of hematochezia. The patient states he has noticed this for 2 days. He states there is a streak of bright blood along his stool every time he has a bowel movement. The patient has no pain with his bowel movements. He admits to eating a poor diet. The patient has no abdominal pain, nausea, or vomiting. On physical exam the patient has a positive fecal occult blood test but has no noticeable rectal bleeding or lesions. What is the likely diagnosis? • Internal hemorrhoid (This is the likely diagnosis, as the patient has painless hematochezia) • External hemorrhoid (This would be painful, and on exam the patient would likely have a sign of an inflamed blood vessel at the rectum) • Bleeding peptic ulcer (This would present with dark bloody stool) • Rectal fissure (This would cause pain every time the patient had a bowel movement) 103. What would you expect to see on an abdominal series that would lead toward a diagnosis of small-bowel obstruction? • Air-fluid levels (This finding on an abdominal x-ray is the hallmark of a small-bowel obstruction) • A lead pipe colon (This finding on an abdominal x-ray would be seen in toxic megacolon, a sequela of inflammatory bowel disease) • Free air under the diaphragm (This would be seen on a bowel perforation) • Steeple sign (This is a narrowing of the esophagus seen in croup) 104. Which gastrointestinal disease below could theoretically be completely eradicated with a total colectomy? NR511 MIDTERM BEST EXAM PRACTICE REVIEW STUDY GUIDE LATEST UPDATE 2022/2023 NR511 MIDTERM BEST EXAM PRACTICE REVIEW STUDY GUIDE LATEST UPDATE 2022/2023 • Crohn’s disease (This disease can be anywhere in the digestive system; therefore, a colectomy would not eradicate the disease) • Irritable bowel syndrome (This is generally a psychological condition that presents with gastrointestinal symptoms. Diet modification, cognitive therapy, and antidepressant medications work well) NR511 MIDTERM BEST EXAM PRACTICE REVIEW STUDY GUIDE LATEST UPDATE 2022/2023 NR511 MIDTERM BEST EXAM PRACTICE REVIEW STUDY GUIDE LATEST UPDATE 2022/2023 • “The virus is causing irritation of the gastrointestinal lining, which causes diarrhea.” (Irritation of the gastrointestinal (GI) lining in itself will probably not cause diarrhea) • “The infectious agent invaded the stomach lining and is affecting the balance of water and nutrients.” (In 80% of cases, gastroenteritis is viral in nature. This viral infection causes diarrhea by stimulating the secretion of electrolytes into the intestine. This is rapidly followed by water along the osmotic gradient, resulting in watery stools) 111. When Sammy asks you what he can do to help his wife, who has dumping syndrome, what do you suggest he encourage her to do? • Eat foods higher in carbohydrates (To help clients with dumping syndrome, suggest that they reduce the amount of carbohydrates consumed) • Eat 3 large meals plus 3 snacks per day (To help clients with dumping syndrome, suggest that they eat 6 small meals per day) • Eat foods with a moderate fat and protein content (To help clients with dumping syndrome, suggest that they eat foods with a moderate fat and protein content. These foods tend to leave the stomach more slowly and do not draw fluid into the intestine) • Drink fluids with each meal (To help clients with dumping syndrome, suggest that they take fluids between meals and not at mealtime) 112. You elicit costovertebral angle tenderness in Gordon, age 29. Which condition do you suspect? • Cirrhosis (Percussion of the abdomen may elicit pain due to cirrhosis) • Inflammation of the kidney (Costovertebral angle tenderness is tenderness or sharp pain that is elicited when one hand is “thumped” with the ulnar edge of the other fist over the 12th rib at the costovertebral angle on the back. It indicates inflammation of the kidney (and possible associated renal calculi, renal artery or vein occlusion, and perirenal abscess) • Inflammation of the spleen (Percussion of the abdomen may elicit pain due to inflammation of the spleen) • Peritonitis (Percussion of the abdomen may elicit pain due to peritonitis) 113. You are trying to differentiate between functional (acquired) constipation and Hirschsprung disease in a neonate. Distinguishing features of Hirschsprung disease include which of the following? • Small ribbonlike stools (Hirschsprung disease is common in male infants, results in small ribbonlike stools, usually has no accompanying abdominal pain unless there is obstruction, and may be accompanied by failure to thrive. Other symptoms may include swollen belly; vomiting, including vomiting a green or brown substance; constipation; gas, which might make a newborn fussy; and diarrhea) • Obvious abdominal pain (The infant with functional (acquired) constipation is usually male and has very large stools and abdominal pain) • Female gender (Hirschsprung disease is more common in male infants) • Small weight gain (Hirschsprung disease may be accompanied by failure to thrive. This is uncommon in infants with functional constipation) 114. You are doing routine teaching with a patient who has a family history of colorectal cancer. You know she misunderstands the teaching when she tells you she will: • Decrease her fat intake (Decreased fat intake is recommended) • Increase her fiber intake (Increased fiber intake is recommended) • Continue her daily use of aspirin (The daily use of aspirin has been shown to decrease the incidence of colorectal cancer as well as dramatically decrease the incidence of metastasis) • Increase her fluid intake (Increasing fluid intake has not been shown to decrease the risk of colorectal cancer. Current recommendations to aid in preventing colorectal cancer include decreased fat consumption, increased fiber consumption, and the daily use of aspirin) 115. Bobby, age 6, has constant periumbilical pain shifting to the right lower quadrant, vomiting, a small volume of diarrhea, absence of headache, a mild elevation of the white blood cell count with an early left shift, and white blood cells in the urine. You suspect: • Appendicitis (Constant periumbilical pain shifting to the right lower quadrant; vomiting following the pain; a small volume of diarrhea; no systemic symptoms, such as a headache, malaise, or myalgia; a mild NR511 MIDTERM BEST EXAM PRACTICE REVIEW STUDY GUIDE LATEST UPDATE 2022/2023 NR511 MIDTERM BEST EXAM PRACTICE REVIEW STUDY GUIDE LATEST UPDATE 2022/2023 elevation of the white blood cell count with an early left shift; and white blood cells (WBCs) or red blood cells (RBCs) in the urine are indications of appendicitis. The WBC count becomes high only with gangrene or perforation of the appendix. The urine may have WBCs or RBCs if the bladder is irritated and ketonuria if there is prolonged vomiting) • Gastroenteritis (Gastroenteritis would result in a large volume of diarrhea) NR511 MIDTERM BEST EXAM PRACTICE REVIEW STUDY GUIDE LATEST UPDATE 2022/2023 NR511 MIDTERM BEST EXAM PRACTICE REVIEW STUDY GUIDE LATEST UPDATE 2022/2023 • Acute pancreatitis (Acute pancreatitis would result in vomiting and a fever in a child) • Rocky Mountain spotted fever (Rocky Mountain spotted fever would present with nausea, vomiting, and fever) 116. Melva, age 63, presents with an acute exacerbation of pancreatitis, and you are going to admit her to the hospital. Which is the most important factor in determining a negative long-term outcome for her? • Age (The older the patient, the greater the possibility of a negative outcome) • Infection (The most important factor in determining long-term negative outcomes for pancreatitis is the presence of infection. Despite best practices, mortality associated with severe acute pancreatitis remains approximately 20% to 25% because of systemic complications) • Pain (Pain should not factor into this outcome) • Length of time between exacerbations (If a patient has many frequent exacerbations without much time between them, he or she also has a possibility of a negative outcome) 117. To differentiate among the different diagnoses of inflammatory bowel diseases, you look at the client’s histological, culture, and radiological features. Mary has transmural inflammation, granulomas, focal involvement of the colon with some skipped areas, and sparing of the rectal mucosa. What do you suspect? • Crohn disease (Crohn disease would show transmural inflammation, granulomas, focal involvement of the colon with some skipped areas, and sparing of the rectal mucosa. The key words are “skipped areas of mucosal involvement.) • Ulcerative colitis (Ulcerative colitis would show acute inflammatory infiltrates, depleted goblet cells, negative cultures, and continuous involvement of the mucosa) • Infectious colitis (Infectious colitis, because of the toxic products released, may induce periportal inflammation, mild hepatomegaly, and low-grade liver enzyme abnormalities, but usually without trophozoites in the liver) • Ischemic colitis (Ischemic colitis seen on colonoscopy reveals segmental inflammatory changes most often in the rectosigmoid and the splenic flexure, where there is more collateral circulation) 118. You suspect that Nikki has a gastroduodenal ulcer caused by Helicobacter pylori and plan to treat her empirically. What medications should you order? • A proton pump inhibitor (omeprazole), tetracycline or amoxicillin, and metronidazole (Flagyl) (All of the drugs listed are used in the eradication of H pylori. Traditional 14-day “triple therapy” with a proton pump inhibitor (omeprazole), tetracycline or amoxicillin, and metronidazole (Flagyl) has consistently produced eradication rates of approximately 95% and is the least expensive therapy) • Bismuth subsalicylate (Pepto-Bismol) and omeprazole (Prilosec) (Both drugs listed are used in the eradication of H pylori, but do not complete the regimen) • Amoxicillin (Amoxil) and omeprazole (Prilosec) (Both drugs listed are used in the eradication of H pylori, but do not complete the regimen) • Clarithromycin (Biaxin) and metronidazole (Flagyl) (Both drugs listed are used in the eradication of H pylori, but do not complete the regimen) 119. Which of the following medications/drugs are not known to cause heartburn or dyspepsia? • Alcohol. • Motrin. • Prednisone. • Tylenol. 120. A 7-year-old male presents with his mother to the urgent care clinic complaining of abdominal pain. He started to complain of pain prior to going to bed; however, it has gotten progressively worse and is now preventing him from sleeping. He is nauseous but hasn’t vomited and didn’t eat dinner due to the pain. The patient appears pale and is complaining of right-sided abdominal pain. His vitals are as follows: blood pressure 130/85, pulse 120, temperature 100.5°F, pulse oximetry 98% on room air. On physical exam he is tender in the right lower quadrant. His complete blood count (CBC) shows a white blood cell count (WBC) of 17.0. What is the patient’s likely diagnosis? • Appendicitis (This is the clinical scenario typical for appendicitis) NR511 MIDTERM BEST EXAM PRACTICE REVIEW STUDY GUIDE LATEST UPDATE 2022/2023 NR511 MIDTERM BEST EXAM PRACTICE REVIEW STUDY GUIDE LATEST UPDATE 2022/2023 127. Marvin, a known alcoholic with cirrhosis, is frequently admitted for coagulopathies and occasionally receives blood transfusions. His wife asks you why he has bleeding problems. How do you respond? • “Occasionally he accumulates blood in the gut.” (An accumulation of blood in the gut would not affect coagulation) NR511 MIDTERM BEST EXAM PRACTICE REVIEW STUDY GUIDE LATEST UPDATE 2022/2023 NR511 MIDTERM BEST EXAM PRACTICE REVIEW STUDY GUIDE LATEST UPDATE 2022/2023 • “There is an interruption of the normal clotting mechanism” (Because of Marvin’s alcoholism and his resulting dietary insufficiencies, there is an inadequate amount of vitamin K in the liver for thrombin to convert fibrinogen to fibrin; thus, the sequence of coagulation is disrupted) • “Long-term alcohol abuse has made his vessels very friable.” (Large numbers of spider angiomas are associated with liver cirrhosis. Excessive drinking also widens blood vessels and increases blood flow to the skin; hence, it can lead to thread veins, a reddened “drinker’s nose,” and rosacea) • “His bone marrow has been affected.” (While vitamin K plays an important role in bone metabolism and the optimization of bone health, the reverse is not true; bone marrow does not affect the amount of vitamin K) 128. Icterus due to hyperbilirubinemia is seen when the serum level of bilirubin is greater than? • 2.5 mg/dL (A bilirubin level greater than 2.5-3 mg/dL is associated with yellowing of the eyes) • mg/dL. • 2.0 mg/dL. • 0.5 mg/dL. 129. Samantha, age 28, is 100 lb overweight and wants to have a gastroplasty performed. In discussing this with her, you explain that by having this procedure she may: • Develop diarrhea (Diarrhea is a common problem after a gastroplasty because of the induced malabsorption) • Lose too much weight (Clients usually do not end up losing too much weight because sometimes the only foods they can tolerate are high-calorie foods (simple sugars), such as ice cream) • Develop hemorrhoids (Losing weight will probably help a hemorrhoid problem) • Vomit after she eats (While Samantha may vomit after she eats, the problem of diarrhea is much more common) 130. You suspect that Harry has a peptic ulcer and tell him that it has been found to be strongly associated with: • Anxiety and panic attacks (Stress-related conditions, such as anxiety and panic attacks, may contribute to and aggravate peptic ulcer disease) • Long-term use of nonsteroidal anti-inflammatory drugs (NSAIDs) (Long-term use of NSAIDs may contribute to and aggravate peptic ulcer disease) • Infection by Helicobacter pylori (About 90% of cases of peptic ulcers have been found to be caused by infection with the bacteria H pylori) • A family history of peptic ulcers (Research studies have been inconclusive regarding the hereditary factor for gastric and duodenal ulcers.) 131. What is the recommended treatment to eradicate a Helicobacter pylori infection? • Ranitidine, amoxicillin, and clarithromycin for 2 weeks (This treatment is missing a proton pump inhibitor (PPI), instead using an H2 antagonist) • Amoxicillin, clarithromycin, and omeprazole for 2 weeks (This is the recommended treatment to eradicate H pylori) • Bismuth, amoxicillin, and clarithromycin for 2 weeks (This regimen is missing a proton pump inhibitor (PPI), instead using bismuth) • Bismuth, doxycycline, metronidazole, and ranitidine (This 4-medication regimen can be used; however, a proton pump inhibitor (PPI) needs to be exchanged for the H2 antagonist) 132. Which of the following is not a risk factor for cholecystitis? • Female gender. • Obesity. • Sickle cell anemia. • Younger age. NR511 MIDTERM BEST EXAM PRACTICE REVIEW STUDY GUIDE LATEST UPDATE 2022/2023 NR511 MIDTERM BEST EXAM PRACTICE REVIEW STUDY GUIDE LATEST UPDATE 2022/2023 NR 511 Davis Edge Skin Review Questions Week 3 A 22-year-old African American female presents to your family practice office complaining of progressive skin discoloration. She is adopted and has no known family history of skin problems. The patient notes nonpalpable patches of skin loss and blanching of her forehead and both hands and feet. It has developed over a period of 6 months and appears to have stopped. It is not pruritic, and there is no erythema or sign of infectious etiology. What is the most likely diagnosis? • Vitiligo- This is the physical description of vitiligo. • Alopecia Alopecia involves hair loss, not skin discoloration. • Addison Disease This condition involves hyperpigmentation of the skin, not hypopigmentation of the skin. • Tinea Versicolor This refers to hypopigmentation of the skin due to a fungal infection and is noticed mostly after sun exposure. Which presentation is most concerning for skin cancer? • Dark pigmentation of 1 solitary nail that has developed quickly and without trauma. This is concerning for acral melanoma • A 1-mm blue, round, nonpalpable discoloration of the skin that has been present since birth without change. This describes a benign blue nevus, common in patients of Asian descent. • A 5-mm black mole with round, regular boarders. This mole is round, regular, less than 6 mm, and without change; it is likely benign. • A 2-mm brown mole that is raised 1 mm but round and regular. This mole is small, regular, minimally raised, and only 1 color; it is likely benign. A 4-year-old male presents to your pediatric clinic with his mother complaining of an itchy rash, mostly between his fingers. This has been going on for multiple days and has been getting worse. The patient recently started at a new day care. On physical exam, the patient is afebrile and has multiple small (1-2 mm) red papules in sets of 3 located in the web spaces between his fingers. He also has signs of excoriation. What is the treatment for this problem? • Permethrin lotion for the patient and also his family members. – This is the treatment for scabies • Cold compresses and hydrocortisone cream 1% twice a day. - This would decrease inflammation but would not cure the scabies. • Over-the-counter Benadryl cream. This would provide itching relief but would not cure the scabies. • Ketoconazole cream. This would treat a fungal infection, not scabies. NR511 MIDTERM BEST EXAM PRACTICE REVIEW STUDY GUIDE LATEST UPDATE 2022/2023 NR511 MIDTERM BEST EXAM PRACTICE REVIEW STUDY GUIDE LATEST UPDATE 2022/2023 A 27-year-old female comes in to your primary care office complaining of a perioral rash. The patient noticed burning around her lips a couple days ago that quickly went away. She awoke from sleep yesterday and noticed a group of vesicles with erythematous bases where the burning had been before. There is no burning today. She is afebrile and has no difficulty eating or swallowing. What test would confirm her diagnosis? • Tzanck smear. This would show giant cells consistent with herpes simplex virus. • Potassium hydroxide (KOH) prep. This is used to diagnose fungal infections. • Exam under a Wood lamp. This is used to diagnose fungal infections. • Sterile culture sent for aerobic and anaerobic bacteria. This would help with bacterial causes of these lesions; a polymerase chain reaction (PCR) would have to be sent to diagnose herpes simplex. Which condition is not included in the atopic triad? • Aspirin sensitivity This is included in the ASA, or Samter, triad, which also includes nasal polyps and asthma. • Asthma This is included in the atopic triad. • Allergic Rhinitis This is included in the atopic triad. • Eczema This is included in the atopic triad. A 16-year-old male presents to your office. He was sent by an orthopedist. He has recently had surgical fixation of a humerus fracture. The patient has been going to physical therapy and has been developing a rash on his arm after therapy that disappears shortly after returning home. He does not have the rash prior to therapy. The patient denies fevers and chills, and his incision is well healed, with no signs of infection. Of note, the patient has been experiencing more hand edema than the average patient and has had edema wraps used at the end of therapy to help with his swelling. The wraps are made of a synthetic plastic material. The rash the patient gets is erythematous and blotchy, not raised; it is on the operative upper extremity. What is the most likely diagnosis? • Contact dermatitis The patient’s history and rash are consistent with a latex or plastic sensitivity due to the edema wraps used in therapy. • Atopic dermatitis The patient’s rash is not consistent with eczema, which is dry and erythematous and usually found in the skin folds and around the eyes. • Seborrheic dermatitis The patient’s rash is not consistent with seborrheic dermatitis, as no greasy yellow scales are present. • Psoriasis Psoriasis is typically described as silvery scales on top of an erythematous, raised base. NR511 MIDTERM BEST EXAM PRACTICE REVIEW STUDY GUIDE LATEST UPDATE 2022/2023 NR511 MIDTERM BEST EXAM PRACTICE REVIEW STUDY GUIDE LATEST UPDATE 2022/2023 Which of the following statements about psoriasis is not true? • Psoriatic lesions are often silvery scales that form over erythematous plaques. This is a general description of psoriasis. • Psoriatic lesions often occur in the folds of the elbows and behind the knees. This is untrue; lesions usually occur on the fronts of the knees, the posterior aspects of the elbows, and the scalp. • People with psoriasis have a greater risk of depression than the average population. This is true; there is a correlation between psoriasis and an increased risk of developing depression. • Psoriasis has a genetic component. This is true; psoriasis has a genetic component and is associated with genetic findings on chromosomes 4, 6, 8, 16, and 17. Which of the following has/have not been linked to the use of isotretinoin? • Elevated liver transaminases. This is listed as a possible adverse reaction to isotretinoin. • Depression, psychosis, and suicidality. This is listed as a possible adverse reaction to isotretinoin. • Benign intracranial hypertension. This is listed as a possible adverse reaction to isotretinoin. • Pancreatitis. This is not an adverse effect of isotretinoin. A 55-year-old landscaper presents to your primary care office complaining of a small skin lesion on his face. The patient states the lesion causes no pain or other symptoms. On physical exam, you notice a small (3 mm) papule that is flesh-colored and irregular. To palpation, the lesion feels hard and like sandpaper. What type of malignancy is this patient at risk for given the appearance of this lesion? • Squamous Cell Carcinoma. The lesion described is an actinic keratosis, which is a premalignant lesion that can progress to squamous cell carcinoma. • Melanoma Melanoma is a type of cancer that arises in melanin-forming cells; the lesion described here is not melanoma. • Basal Cell Carcinoma Basal cell carcinoma typically presents as a papular lesion with telangiectasia. • Rosacea Rosacea is not associated with cancer. An eczematous skin reaction may result from: • Penicillin Penicillin, neomycin, phenothiazines, and local anesthetics may cause an eczematous type of skin reaction. • Allopurinol (Zyloprim) Allopurinol (Zyloprim) and sulfonamides may cause exfoliative dermatitis. NR511 MIDTERM BEST EXAM PRACTICE REVIEW STUDY GUIDE LATEST UPDATE 2022/2023 NR511 MIDTERM BEST EXAM PRACTICE REVIEW STUDY GUIDE LATEST UPDATE 2022/2023 • Oral contraceptives. Oral contraceptives may cause erythema nodosum. • Phenytoin (Dilantin) Phenytoin (Dilantin) and procainamide (Pronestyl) may cause drug- related systemic lupus erythematosus. NR511 MIDTERM BEST EXAM PRACTICE REVIEW STUDY GUIDE LATEST UPDATE 2022/2023 NR511 MIDTERM BEST EXAM PRACTICE REVIEW STUDY GUIDE LATEST UPDATE 2022/2023 is essential when treating pediculosis. Clients should be informed that itching may continue for up to a week after successful treatment because of the slow resolution of the inflammatory reaction caused by the lice infestation. • “Spraying of pesticides in the immediate environment is essential to prevent recurrence.” Excessive decontamination of the environment is not necessary. Environmental spraying of NR511 MIDTERM BEST EXAM PRACTICE REVIEW STUDY GUIDE LATEST UPDATE 2022/2023 NR511 MIDTERM BEST EXAM PRACTICE REVIEW STUDY GUIDE LATEST UPDATE 2022/2023 pesticides is not effective and, therefore, is not recommended. Bedclothes and clothing should be washed in hot, soapy water. Tom, age 50, is complaining of an itchy rash that occurred about a half hour after putting on his leather jacket. He recalls having a slightly similar rash last year when he wore his jacket. The annular lesions are on his neck and both arms. They are erythematous, sharply circumscribed, and both flat and elevated. His voice seems a little raspy, although he states that his breathing is normal. What is your first action? • Order a short course of systemic corticosteroids. All the actions are appropriate. However, it is most important to determine if respiratory distress is imminent; if it is, epinephrine must be administered. • Determine the need for 0.5 mL 1:1000 epinephrine subcutaneously. Tom has hives. Although all the actions are appropriate, the first step is to determine the need for 0.5 mL 1:1000 epinephrine subcutaneously. With Tom’s neck involvement, it is most important to determine if respiratory distress is imminent; if it is, epinephrine must be administered. • Start daily antihistamines. All the actions are appropriate. However, it is most important to determine if respiratory distress is imminent; if it is, epinephrine must be administered. • Tell Tom to get rid of his leather jacket. All the actions are appropriate. However, it is most important to determine if respiratory distress is imminent; if it is, epinephrine must be administered. When palpating the skin over the clavicle of Norman, age 84, you notice tenting, which is: • Indicative of dehydration Skin turgor is decreased with dehydration. • Common in thin older adults Tenting—which occurs when pinched skin remains pinched for a few moments before resuming its normal position—over the clavicle is common in thin older adults. Skin turgor is decreased with dehydration and increased with edema and scleroderma. • A sign of edema Skin turgor is increased with edema. • Indicative of scleroderma Skin turgor is increased with scleroderma The nurse practitioner (NP) tells Samantha, age 52, that she has an acrochordon on her neck. What is the NP referring to? • A nevus- a nevus is a mole • A skin tag- Skin tags (acrochordons) are benign overgrowths of skin commonly seen after middle age and usually found on the neck, axillae, groin, upper trunk, and eyelids. • A lipoma A lipoma is a benign subcutaneous tumor that consists of adipose tissue. • A wart A wart is a circumscribed elevation due to hypertrophy of the papillae and epidermis. NR511 MIDTERM BEST EXAM PRACTICE REVIEW STUDY GUIDE LATEST UPDATE 2022/2023 NR511 MIDTERM BEST EXAM PRACTICE REVIEW STUDY GUIDE LATEST UPDATE 2022/2023 Maryann, age 28, presents to the clinic because of a rapid onset of patchy hair loss. The skin within these oval patches of hair loss is very smooth. Tapered hairs that resemble exclamation points are seen at the margin of a patch of hair loss. Based on these findings, you suspect Maryann has: • Alopecia areata The findings are consistent with alopecia areata, ie, nonscarring hair loss of rapid onset, the pattern of which is most commonly sharply defined round or oval patches. • Trichotillomania The cause of trichotillomania is mechanical, and the patch typically has an irregular, angulated border. • Tinea Capitis Tinea capitis is caused by a fungal infection of the skin and hair shaft. Most commonly there is diffuse or patchy adherent scale on the scalp. • Androgenetic alopecia Androgenetic alopecia is premature loss of hair in an androgen- sensitive area of the scalp; in men, it is commonly known as male-pattern baldness. Roy, age 13, was recently diagnosed with epilepsy and prescribed carbamazepine for control of his seizures. He has developed erythematous papules, dusky appearing vesicles, purpura, and target lesions that have erupted rapidly and are more centrally distributed on the face. He has hemorrhagic crusts on his lips. He tells you his skin feels tender and burns. Additionally, he has developed exudative conjunctivitis. These findings are indicative of: • Urticaria Exudative conjunctivitis is not a typical finding in urticaria. • Pemphigus vulgaris Exudative conjunctivitis is not a typical finding in pemphigus vulgaris. • Herpetic gingivostomatitis Exudative conjunctivitis is not a typical finding in herpetic gingivostomatitis. • Steven Johnson Syndrome SJS is a severe blistering mucocutaneous syndrome that involves at least 2 mucous membranes. Drugs frequently implicated in the development of SJS are phenytoin, phenobarbital, carbamazepine, sulfonamides, and aminopenicillins. Eric, age 52, has gout. What do you suggest? • Using salicylates for an acute attack. Salicylates should be avoided because they block renal excretion of uric acid. • Limiting consumption of purine-rich foods. For a client with gout, the consumption of purine- rich foods, such as organ meats, should be limited to prevent uric acid buildup. • Testing his uric acid level every 6 months. Annual testing of the serum uric acid level is sufficient. • Decreasing fluid intake. Fluids should be increased to 2 L per day and alcohol should be limited. NR511 MIDTERM BEST EXAM PRACTICE REVIEW STUDY GUIDE LATEST UPDATE 2022/2023
Docsity logo



Copyright © 2024 Ladybird Srl - Via Leonardo da Vinci 16, 10126, Torino, Italy - VAT 10816460017 - All rights reserved